Anda di halaman 1dari 80

Historical Considerations

We must never forget, it is a constitution we are expounding. Chief Justice John Marshall
Allocation of Power

Horizontal Allocation of Power


o Separation of power into three branches

Article I: Legislative (Congress)

Article II: Executive (President)

Article III: Judicial (Supreme Court and Federal Courts)

Vertical Allocation of Power


o Federalism

Federal Government and the Several States


Constitution v. Statute

Durability Statutes are easy to add, amend, or repeal; Constitutions are not
o Only one Constitution since 1789; Only 27 Amendments (10 of which passed at the same time)

Breadth Constitutions are broad in scope; Statutes pinpoint small issues

Anti-majoritarian The Constitution is designed to prevent majority tyranny; Statutes represent the
will of the majority
o The Constitution protects the minorities who may easily be subjected to unfair treatment by
the majority as it exerts its voting power.

Supremacy Clause Constitution is the supreme law of the land. All other laws must be consistent
with it, or they are void.
Amending the Constitution

Congressional Initiative
o 2/3 approval of both houses
o of states ratify

2nd Constitutional Convention


o Has never happened, and never will
o Everything is on the table to potentially be changed. Nothing would be safe.
Principle of Decision-Making Judicial Restraint (Ashwander)

Brandeis once said Courts should decide issues on sub-constitutional considerations whenever possible,
so that the legislatures may more easily overrule the courts decision by changing or fixing the
deficiency in the statute.
Bill of Rights

Baker v. Carr (J. Jackson) The purpose of the Bill of Rights was to remove certain subjects from the
political sphere, placing them out of reach of the majority and government officials. These rights are
fundamental and may not be made subject to a vote.

Legitimacy of Judicial Review

Article III - Generally


o Creates the Supreme Court of the United States, and permits Congress to establish lower
federal courts that are vested with Article III power
o Article III ensures the independence of federal judges

Federal judges have tenure, and their salary cannot be reduced


o Article III never expressly grants power to review constitutionality of statutes (federal or
state)

Article III, 2 The Federal Judicial Powers


o Art. III, 2 sets out the federal judicial powers

Cases arising under the Constitution or federal laws (federal question)

Cases of admiralty

Cases between two or more states

Cases between citizens of different states (diversity jurisdiction)

Cases between a state or its citizens and a foreign country or citizen

Marbury v. Madison
o Issue: Do the courts have the authority to declare a statute unconstitutional (and therefore
void) if it concludes the statute violates the Constitution?
o Holding: Yes. The courts have the power of judicial review, and may review acts of the
Executive and Legislative branches and deem those acts void as violative of the Constitution.

Judicial Review Today


o Supremacy Clause = Federal Law is Supreme
o Courts may interpret the Constitution in the cases before them
o Judicial interpretation is final and controlling for all purposes, not just the pending case
o The political branches may overrule the Courts constitutional decision by constitutional
amendment they may overrule a statutory decision by statutory amendment

Additional Notes on Legitimacy of Judicial Review


o This doctrine did not provoke much opposition because it was rarely used early on

It was not applied to a federal statute again until Dred Scott, 50 years later
o Judicial Review has become a major stabilizing force in American Law
o We ask judges to be the people we call upon to interpret the Constitution, and to tell people
they cant have everything they want merely because they have a majority vote.
o Judicial review influences how lawmakers create their laws get it right the first time, so it
doesnt get ruled unconstitutional
o These judges are not elected officials, which often causes tension because they have the ability
to overrule democratically enacted laws

Congressional Control of Judicial Review

Two ways Congress can Control Article III


o Jurisdiction-Removal Bills

Power to strip SCOTUS of its appellate court jurisdiction under the exceptions and
regulations clause
Exceptions Clause: the supreme Court shall have appellate Jurisdiction, both as to Law
and Fact, with such Exceptions, and under such regulations, as the Congress shall
make.

Example of Attempted J-Removal Bill:

Pledge Protection Act of 2005 Attempted to prevent SCOTUS from reviewing


the validity of the Pledge of Allegiance under the Constitution

Constitutionality of Jurisdiction-Removal Bills

Jurisdiction-removal bills are of dubious constitutional validity

Ex Parte McCardle

Held: Congress may exercise its Article III power to decide what types
of cases SCOTUS may hear, but it may not expand the federal judicial
power

US v. Klein

Held: Congress may not utilize Article III power of jurisdiction-removal


to preclude a colorable claim made under another provision of the
Constitution.

Example: Congress may not prevent a citizen from brining a suit


alleging the pledge of allegiance violates the 1st Amendment, because
the claim is made under the 1st Amendment
o Constitutional Amendment

This would require the passage of an amendment declaring a certain issue off-limits for
SCOTUS interpretation
Congressional Control of the Jurisdiction of Lower Federal Courts
o Congress, as created of the lower federal courts, may decide which lower federal courts may
exist, and what cases they may hear.
o However, they may not allow the lower courts to hear cases outside of the federal judicial
power granted in Article III
Additional notes on Congressional Control of Judicial Review
o Congress can give jurisdiction where it did not previously exist

Example: Terry Schiavo case


o Courts have the power to determine their own jurisdiction. So even if a jurisdiction-removal bill
were passed, the courts have the power to determine whether that bill was constitutionally
valid.
o Why are so few Jurisdiction-Removal bills passed?

Dubious constitutional validity

Passions fade and cooler heads prevail

Concerns about freezing the law when courts cannot interpret it

State courts can still decide the case, which may be problematic in some situations

Federal Review of State Court Decisions

Rule

Supreme Court may review state court decisions, but only to the extent that the decision
was based on federal laws.
o Review is limited to decisions of the highest state court that made a decision

If intermediate court issues opinion, and state supreme court refuses to hear case,
SCOTUS may review the intermediate courts decision
o Note the Adequate and Independent State Grounds Doctrine
History
o Article III and the Judiciary Act of 1789

Congressional act provided for Supreme Court review of state court judgments
o Martin v. Hunters Lessee

Held:

Appellate power of the United States extends to state court judgments

SCOTUS has jurisdiction to review the final decisions of the state courts

This is an example of vertical allocation of judicial power

J. Story used the Supremacy Clause to justify determination that federal courts
may determine whether state laws are consistent with the Constitution
o Cohens v. Virginia: Extended holding of Martin to apply to the decisions of state courts in
criminal cases a federal issue must still be implicated.
Additional Notes on Federal Review of State Court Decisions
o Following Marbury and Martin, SCOTUS has the power to hear cases coming from federal and
state courts.
o However, SCOTUS may only review issues of federal law
o SCOTUS has authority to review the decisions of the highest state courts where those decisions
implicate the federal constitution, a federal statute, or a federal treaty.
o SCOTUS has no jurisdiction to review a state court decision unless a federal issue is implicated.
o

Adequate and Independent State Grounds Doctrine

This doctrine is a limitation on the ability of SCOTUS to review a state courts decision

Rule
o Even if there is a federal question in a state court case, SCOTUS may not review the state
courts decision if there is an adequate and independent state ground for the decision

Application of the Rule


o Two Branches of Analysis

Adequate (Procedural)

The nonfederal grounds for the decision must be adequate (dispositive) of the
case. Such that even if the federal issue in the case was wrongly decided,
reversal on the federal issue would not change the outcome of the case.

Independent (Substantive)

If the state courts interpretation of the state provision was based on


interpretations of federal law, the state law grounds are not independent, and
SCOTUS may review

The state law provisions and their interpretations must be entirely independent
of federal law and its interpretations

General Explanation of the Rule


o If a state court rules against a party on two alternative grounds, one federal and one state, the
Supreme Court lacks jurisdiction to review the decision if the state courts judgment rests
upon an adequate and independent state ground.

Additional notes regarding Adequate and Independent State Grounds Doctrine


o Determination that a state law is consistent with both the state and federal constitution does
not preclude review SCOTUS may review to determine the statutes validity in the face of
federal constitution
o If reversal of the state courts determination as to the federal grounds would change the result
of the case, there are no adequate and independent state grounds
o If lawsuit is based on US Constitution, but state court decides based on state constitution,
SCOTUS may review because the decision relied on interpretation of the federal constitution
o Where state court, in bad faith, does not rule on the federal issue that was properly raised,
SCOTUS may review if determination of the federal issue would change the result of the case.
Doctrine of Justiciability in General

Definition
o Limitation on the ability of federal courts (and SCOTUS) to hear cases.
o If the court cannot hear a case due to the restrictions within this doctrine, the case or issue is
nonjusticiable

Considerations behind Justiciability


o Art. III, 2, Cl. 1 - Cases and Controversies Requirement

Federal Courts may only hear cases and controversies


o Separation of powers

Notion that a decision in the case might unduly intrude on the other branches of
government
o Policy (Prudential) Considerations

Preserve judicial resources for matters most deserving of review

Make better decisions, because those decisions are based on real controversies

Fairness, especially for individuals not before the court

Achieve balance between restraint and review

5 Sub-Doctrines of Justiciability
o Advisory Opinions
o Standing
o Mootness
o Ripeness
o Political Questions

Additional Notes
o These doctrines generally assume that subject matter jurisdiction and personal jurisdiction
exist
o Activist Court: Expand constitutional doctrines and narrow justiciability limitations
o Retrenchment Court: Tend to avoid reaching merits of case by construing justiciability
limitations more broadly
o A decision made on justiciability grounds never decides the case on its merits
o Those circumstances preventing justiciability may never be waived by the parties

Advisory Opinions

In General
o Art. III, 2, only allows jurisdiction over cases and controversies
o Federal Courts are prohibited from giving an opinion when no party before the court has
suffered or faces a specific injury

Rule

For a case to be justiciable, and not an advisory opinion, two criteria must be met:

1. There must be an actual dispute between the parties

2. There must be a substantial likelihood that a decision in favor of the claimant will
bring about some change or have some effect.
Examples
o Executive branch doubts a bill, and asks Court to determine bills constitutionality
o Collusive Suits The two parties appear to oppose one another, but actually seek the same
outcome
Differences from a Declaratory Judgment
o A DJ is a determination as to the legal positions of the parties when there is doubt as to their
positions

These resolve an actual case or controversy by providing legal certainty to each party
o Suits for declaratory judgment are justiciable so long as they meet the requirements for judicial
review (may not be hypothetical or abstract)
Flast v. Cohen
o The requirement of a case or controversy limits the questions of a case to those that may be
resolved through the judicial process
o Rule: SCOTUS cannot answer abstract questions posed to it. If the question is outside the
scope of the issue, the court cannot answer it.
Additional Notes on Advisory Opinions
o Some states permit advisory opinions in some situations
o Dicta does not constitute an advisory opinion, and is not prohibited by Article III

Article III is not concerned with what might be put into an opinion, but the entire
posture of the case

One might say that dicta should be discouraged from a prudential standpoint
Missouri allows advisory opinions of its state supreme court
o

Standing

Rule (Dont forget Prudential)


o Constitutional Standing

Injury-in-Fact Plaintiff must allege that is has personally suffered an imminent


threatened or actual injury. It must be a distinct and palpable injury to himself.

Causation The alleged injury must be fairly traceable to the defendants conduct
Ds conduct must be the cause of the harm.

Redressability Must be likely that a favorable court decision will redress the plaintiffs
injury.
o Statutory Standing

If the case involves a cause of action created by statute, the plaintiff must be within
the class of people identified by the statute as a proper plaintiff.

Ex: If statute says spouses may sue for harm done to worker in workplace, and
the worker only has a girlfriend, the girlfriend lacks standing because she is not
his spouse.

Standing Analysis
o Does Plaintiff have Article III standing? If no, dismiss.

Injury, cause, redressability

o If yes, are there prudential considerations that might bar standing?


o Does the plaintiff have sub-constitutional standing (through statute or common law)?
o Consider the exceptions to the prudential concerns.
In General
o Standing is a pre-trial decision. It never gets to the merits of the case.
o Courts may use standing as a way of avoiding hard issues.

10

10

If Constitutional Standing requirements are met, one of the prudential considerations for
standing may come into effect and prevent standing.
Injury-in-Fact Analysis
o Plaintiff must show that he himself has been injured by the conduct of which he complains, OR
that he will probably suffer some concrete injury if the conduct is allowed to occur.
o Individuated Harm: The injury cannot merely be the same as that suffered by every citizen
or taxpayer.
o Non-Economic Harm is sufficient to establish an injury
o Violations of statutory rights establish an injury-in-fact

Requirement: Injury; Infringement of statutory right


o Injuries to Common Law Rights are sufficient for standing purposes

Ex: Property, contracts, torts, etc


o Injuries to Constitutional Rights

Two important questions

1. What Constitutional Provision bestows the right?

If a violation of individual liberties, then there will be standing

If a violation of provision dealing with structure of government,


then there likely will not be standing unless P can show a distinct
personalized harm

2. Do that facts demonstrate an injury?


Causation and Remedy Analysis
o Did the defendants action cause the harm (or will it cause the harm)?
o Can a favorable court decision redress the injury suffered by the plaintiff?

Differentiate between prospective relief (injunction) and retrospective relief (damages)


o

MAKE SURE YOU ADDRESS PRUDENTIAL STANDING!!!!!

11

11

12

12

Prudential Standing Considerations

Even what Article III standing exists (or might exist), the courts may refuse to hear a case out of
prudence

There are two main types of prudential standing considerations


o Third-Party Standing
o Generalized Grievances

Taxpayer Suits

Citizen Suits

Suits by Government Officials

Third-Party Standing
o General Rule: A party with Article III standing may not argue the rights and liabilities of third
parties not before the court. The party may only argue its own legal rights.
o Purpose: Improve the quality of litigation by ensuring that those litigating have something
personally on the line (their own rights and liabilities); Individuals are the best proponents of
their own rights; Third-persons may be bound by a judgment even though they never litigated
the issue
o Exceptions

1. Where a close relationship exists between plaintiff and third-parties

Examples: Craig v. Boren (vendor/purchaser); Glucksberg (physician/patient);


Pierce (school/parents); Mass. v. EPA (state sues on behalf of citizens)

2. Where an association sues on behalf of its injured members

Associations may come in so long as they or their members can establish


Article III standing the association may then argue the rights and obligations
of its members

3. Where the third-party is unlikely to be able to sue

Ex: Craig v. Boren (kids would age out of case and make it moot)

13

13

A less important exception today, because this litigation is heavily funded by


interest groups who find the perfect plaintiff

4. First Amendment Overbreadth Doctrine

Even if a speech statute is applicable to the individuals conduct, the individual


may argue that the statute is overbroad and encroaches on protected forms of
speech. The court may hear the case and strike the statute down for being
overbroad, even if it was constitutional as applied to the individual.
Notable Case Craig v. Boren

19 year old Craig and saloonkeeper sue Oklahoma for discriminatory drinking laws
(gender discrimination girls = 18, boys = 21)

Craig was mooted out of case when he turned 21

Saloonkeeper wants to maintain action, even though she would be arguing the rights of
men under 21 rather than her own rights

Saloonkeeper had Article III standing (loss of money, statute was case of harm, and
injunction would remedy)

Because of saloonkeepers connection to the harmed party in this case (men under 21
who want to buy alcohol), she could proceed with the suit

Generalized Grievances
o Taxpayer Suits

Plaintiff claims injury on the basis that they are paying taxes, and their taxes are being
use improperly.

As a general principle, taxpayers interests are too remote to allow them to proceed
with a suit, even if they have Article III standing. (Frothingham v. Mellon)

Exception Flast v. Cohen

Taxpayer suits are permitted when

1. The taxpayer claims unconstitutional use of federal funds.

2. The taxpayer challenges the spending based on a specific


Constitutional limitation on the taxing and spending power.

In Flast, the taxpayers alleged that government was unconstitutionally


providing funding to religious schools, in violation of establishment clause
(which is a specific constitutional limitation on the taxing and spending
power).

Flast has never been expanded beyond establishment clause cases.

Further, Flast only applies to congressional spending, not executive spending


(Hein).
o Citizen Suits

Occurs when a plaintiff claims that he can challenge the constitutionality of an act
based on the fact he is a citizen

This is a generalized grievance, in that everyone is impacted equally

14

14


No standing for prudential reasons
Rule: Plaintiff cannot seek relief that no more directly or tangibly benefits the plaintiff
than it does the public at-large

A citizen suit may meet Art. III standing requirements, but not prudential concerns

Merely arguing standing on the basis of being a citizen will not be enough

Lujan v. Defenders of Wildlife

Congress may allow for citizen standing through legislation, HOWEVER,


Congress cannot overcome the Art. III standing requirements through
legislative action

There, although the citizens had statutory standing, they lost on the basis that
they lacked a concrete injury-in-fact, and therefore had no Article III standing
to sue.

FEC v. Akins

Held: Citizen had standing to sue when they alleged violation of law that
required group to file documents. The citizens had a specific injury in that they
had a right to see the document, but no document existed because the statute
was violated.

Hollingsworth v. Perry

Held: Citizens who were Official Proponents of Proposition 8 lacked standing


to defend the provision. They were not injured by states refusal to defend the
suit and could not stand in for the state.
Government Official Suits

Sometimes government officials sue because they believe a law is unconstitutional

Government officials cannot sue unless they have a discrete, personal injury in the
case

Example: House Rep. sue when Congress would not seat him simply because
they disliked him. He claimed act was unconstitutional. Held to have standing
because he was personally injured.

Example: An education official is required by law to provide textbooks to


religious schools. He has standing to sue because his job would be at stake if
he refused to follow law when he believed it was unconstitutional.

15

15

16

16

Mootness

Definition
o A case is moot if it raised a live controversy at the time of the complaint, but events
occurring after the filing have deprived the litigant of an ongoing stake in the controversy.

Common example: Student sues school for violating establishment clause. Student
graduates while case goes through courts. Case is now moot because student no longer
has a stake in the controversy.

Mootness v. Standing
o Standing is a pre-trial issue
o Mootness is considered throughout. It is essentially the standing doctrine exercised throughout
the course of the case.

Relief sought is crucial


o In Craig v. Boren, if saloonkeeper pursues damages, case will never become moot. If she seeks
an injunction, then case will become moot if legislature changes the law to eliminate the
discrimination.

Examples of Moot Cases


o Legislatures changes or repeals law at issue in case

Can beat this by pursuing damages instead of just an injunction


o Criminal defendant dies, or civil plaintiff dies and cause of action does not survive death
o Prosecutor drops charges against criminal defendant
o Civil parties settle case
o Student sues, but graduates before resolution

Can beat this by pursuing damages instead of just an injunction

EXCEPTIONS to the Mootness Doctrine


o Capable of repetition, yet evading review

Applies when: Injury is of such a brief duration that the plaintiff would almost always
suffer it before the federal court litigation is completed.

Rule: There must be a reasonable expectation that the same plaintiff could be
subject to the same wrong again.

Example: Roe v. Wade Roe gave birth before case concluded, but it met this
exception because Roe could have become pregnant again, and been subject to
the same restrictions.
o Collateral Injuries

Applies when: The claim itself is mooted, but there is some collateral consequence to
the harm that continues

17

17

Rule: A case is not moot if there are still collateral consequences that might be
adverse to the party.

Example: Criminal D serves entire sentence before appeals conclude, but the
conviction on his record can still harm him.
Voluntary Cessation of Conduct by Defendant

Applies when: P sues to obtain injunction against unlawful conduct, and the D ceases
the conduct. However, the D is capable of resuming the conduct in the future.

Rule: When seeking an injunction against conduct, Ds voluntary cessation of conduct


will not moot the case, unless D shows there is no reasonable likelihood that he will
continue conduct in the future.

Example: Erie v. Paps AM D owned topless bar in violation of city ordinance.


City sues to get it shut down, D retires and closes bar. Held: Ds advanced age
does not make it clear that he will not open another bar in the future, so the
case is not moot.
Certified Class Actions

So long as a there is a member of the class that has not been mooted out of the case,
the case may go forward.

Simply need to replace the named plaintiff when the named plaintiff is mooted from the
case.

18

18

Ripeness

A case is not yet ripe if it has not yet become sufficiently concrete so as to be easily adjudicated.
o Dismissal of the case on the basis that the facts are not clear enough to identify a dispute.

Weighing the Interests


o Hardship to the parties from withholding judicial decision now
o Fitness of the issues for judicial review now

Hardship
o Sometimes, dismissal for ripeness puts one of the parties in a position that it must violate the
law before the case can be heard. This can result in a loss of job or reputation.

19

19

Mitchell Group of plaintiffs said Hatch Act violates freedom of speech. Some plaintiffs were
fired for engaging in political activities, others wanted to engage in the activity, but had not yet
been fired.

Held: Those who have been fired may maintain the suit. Those who have not been
fired do not have a ripe case.
o Adler States passed a subversive organizations statute that made membership in certain
groups a basis for firing of public officials. Teachers filed suit under the 1st Amendment.

Held: SCOTUS took the case, saying it was sufficiently ripe, even though no one joined
the organization and had not yet been fired.

Reason: People should not be forced to exercise their rights at the peril of criminal
sanctions or loss of employment
o Mitchell and Adler are clearly inconsistent with each other
Uncertain Enforcement of Criminal Statutes
o When P attacks constitutionality of a statute he violated, but it is clear that the statute will not
be enforced against him, the case is not ripe.
Difference between Ripeness and Standing
o When a case is dismissed for lack of standing, the plaintiff is not the proper person to bring
the case.

If youre not the right plaintiff today, you wont be the right plaintiff tomorrow.
o When a case is dismissed for ripeness, then the plaintiff is the right person to bring the case,
but the facts have not become ripe enough to allow for judicial determination.

The plaintiff can bring the case when the facts develop.

Statute of Limitations tolls during the period the court says the case is not ripe.
o

20

20

Political Questions

These are issues that the court believes should be left to the legislative and executive branches, even
though the court may have Article III jurisdiction.
o Example: Lawsuit claims Vietnam war was unconstitutional because there was no declaration of
war.

Difference between Political Questions and Standing


o Standing goes to the nature of the plaintiff
o Political questions go to the nature of the claim
o In a political question issue, the court will not decide the issue at all. In a standing case, the
court might decide it for someone else.

Commitment to Other Branches


o Court declines to hear case because it might infringe on the separation of powers.
o There is a textually demonstrable constitutional commitment of the issue to another branch
of government.
o Examples:

What constitutes a declaration of war

Impeachments Impeachments are non-justiciable because they are a method of


removing judges, and courts should not rule on something that may affect the judges
who sit on them.

Lack of Judicial Manageable Standards


o There are no judicially discoverable standards by which to judge the issue

Example: Lawsuit claiming that Rhode Island lacked a republican form of


government. Court found no standard for making that determination. (Luther v.
Borden)

Unsuitable Policy Determination


o A judicial decision would involve making policy determinations that reside in the realm of the
political branches

Reapportionment
o Reapportionment cases are justiciable under equal protection. All voters must have essentially
the same voting power. (Baker v. Carr)
o

Political Question Test:


Textually demonstrable commitment of the issue to a coordinate political
department
Lack of judicially discoverable and manageable standards for resolving it;
Impossibility of deciding without an initial policy determination of a kind clearly
for nonjudicial discretion;
Impossibility of a courts undertaking independent resolution without expressing
lack of the respect due coordinate branches of government;
An unusual need for unquestioning adherence to a political decision already
made; OR
The potentiality of embarrassment from multifarious pronouncements by various
departments on one question.

21

21

Sources of Constitutional Liberties

The original constitution

The bill of rights

The civil war amendments


Rejection of Incorporation Pre-Civil War

Incorporation of the Bill of Rights against the States did not occur until the adoption of the 14th
Amendment

Barron v. Baltimore
o Held: Bill of Rights only applies against the federal government, not the States
The Civil War Amendments

13th Amendment Abolishes slavery

14th Amendment Privileges and Immunities Clause; Due Process Clause; Equal Protection Clause

15th Amendment Grants right to vote (cannot be denied based on race)


Privileges and Immunities Clause and The Slaughterhouse Cases

Clause: No State shall make or enforce any law which shall abridge the privileges and immunities of
citizens of the several states.

Slaughterhouse Cases
o Facts: Hundreds of butchers operated in New Orleans. State passed a reasonable health
regulation, then gave a monopoly to one slaughterhouse. Plaintiffs, several butchers, brought
suit challenging the monopoly as violating their right to practice their trade.
o Held: Privileges and immunities clause not meant to protect individuals from state government
actions, and was not meant to be a basis for federal courts to invalidate state laws.
o This case has never been overruled, and this has precluded the privileges and immunities
clause from being applied to the Bill of Rights
o This case provided a list of rights that were protected under the Constitution under this clause.

22

22

o This interpretation was intended to rob the clause of all of its meaning.
Revival of the Privileges and Immunities Clause
o Saenz v. Roe Privileges and Immunities Clause protects the right to travel and the citizens
right to be treated equally in her new state of residence.

Incorporation

Debate
o Total Incorporationists believed the Bill of Rights should be fully incorporated via the due
process clause
o Selective Incorporationists said only some amendments should be incorporated.
o Selective incorporation won out; Bill of Rights was incorporated one amendment at a time,
as the circumstances and cases came before the Court.

Palko v. Connecticut
o Dealt with incorporation, but it has since been overruled
o Involved double jeopardy
o Created a test for incorporation: Was the right implicit in the concept of ordered liberty/

Duncan v. Louisiana Current law for incorporation


o Held: Amendment is incorporated and applicable against the States by the 14th Amendment if
it is fundamental to the American scheme of justice.

Incorporation Today
o 1st Amendment = Fully Incorporated
o 2nd Amendment = Fully Incorporated
o 3rd Amendment = No SCOTUS decision
o 4th Amendment = Fully Incorporated
o 5th Amendment = Incorporated except for clause guaranteeing grand jury indictment
o 6th Amendment = Fully Incorporated
o 7th Amendment = Not Incorporated
o 8th Amendment = Cruel and Unusual Punishments Clause Incorporated; no ruling on
excessive fines or excessive bail clauses
o Unique exception: 6th Amendment requires 12 person juries, but the states are permitted to
use as few as 6

Why have duplicative clauses in state constitutions?

23

23

States can provide greater protections with their own constitutional provisions than those
provided by the federal constitution.

Practical Usage
o When arguing an incorporated amendment, you can argue using federal precedent even when
in state court, because that precedent is applicable against the state.
o
Due Process

5th and 14th Amendments: Neither the United States not the state governments shall deprive a person
of life, liberty, or property without due process of law.
o

Substantive Due Process vs. Procedural Due Process

Procedural Due Process


o This is a claim that one did not receive the constitutionally mandated process. It does not
challenge the action, but the procedure by which the action was taken.

The government has the right to take the action, but violated procedural requirements
in taking the action.
o Examples: Notice, opportunity to be heard, impartial decision-maker

Substantive Due Process


o This claim challenges the sufficiency of the government decision itself. This requires a
sufficient justification for the governments action
o Sufficiency of the justification depends on the level of scrutiny applied
o In substantive cases, the issue is what the government is restricting.
o In equal protection cases, the issue is who the government is restricting.

Remedy Sought
o Substantive: Plaintiff seeks to have action declared unconstitutional as violating a
constitutional right.
o Procedural: Plaintiff seeks to have action declared unconstitutional because of the lack of
adequate safeguards.

Important Concepts
o It is necessary to understand and define liberty and property for due process
considerations

Two Major Forms of Substantive Due Process


o Economic Substantive Due Process
o Personal Liberties Substantive Due Process

24

24

Economic Due Process Pre-Lochner

Slaughterhouse Cases deny use of Privileges and Immunited Clause to protect substantive rights

Industrialism develops, causing major advancements in economic thought and application of industry
to production of good

Substantive due process emerges as an idea in Munn v. Illinois (1876)


o Rights of property cannot be taken away without due process

Increase in industry and production creates calls for regulation by the government for the purposes of
the public welfare, morals, and standards of living

Regulation of business led to challenges to that regulation

Allgeyer v. Louisiana
o Facts: State law prohibited payment on insurance policies issued by out-of-state companies no
licensed to operate in the state
o Held: Law interferes with freedom of contract
o Significance: The Court invalidated a regulatory law as violating the substantive due process
rights of parties to a contract
Economic Due Process Lochner Era

Lochner v. New York


o Facts: State law set maximum hours that bakers could work. Intended to protect the health of
bakery workers.
o Held: Law violates substantive due process because it interefered with freedom of contract
and because it did not have a valid police purpose

Major Principles of the Lochner Era


o Freedom of contract is a basic right protected as a liberty and property right
o Government may interfere with freedom of contract only to serve a valid police purpose:
protect public safety, health, and morals
o Judiciary had role of carefully scrutinizing legislation interfering with freedom of contrct

This was the origin of classic substantive due process analysis


o Ensure that the laws serve an adequate purpose in restricting a freedom

Lochner Test
o Is this a fair, reasonable, and appropriate exercise of the police power of the state, or is it an
unreasonable and arbitrary interference with the right of individuals to exercise liberty?

After Lochner (pre-1937)


o Numerous statutes were struck down as violating freedom to contract

25

25

o
o

Those involving women tended to be upheld because women were seen as the weaker sex
that needed to be protected (Muller v. Oregon)
Regulations on business were often seen as interfering with the rights of businesses to act
freely and to form agreements

Post-Lochner Overturned in 1937

Nebbia v. New York (suggests demise)


o Held: Upheld regulation that set prices of milk
o Questioned the basic premise of Lochner

Court declared a need for judicial deference to legislative enactments

Questioned notion that government can only regulate to serve the police purpose

West Coast Hotel v. Parrish (Overturns Adkins, which struck down a minimum wage law)
o Held: Upheld a state law requiring a minimum wage for women employees
o Court declares

Freedom of contract no longer protected as a fundamental right

Government can regulate to serve any legitimate purpose

Judiciary will defer to legislature so long as it is reasonable

US v. Carolene Products Co
o Held: Economic regulations should be upheld so long as they are supported by a
conceivable rational basis, even if that rational basis was not the legislatures actual intent
o Creation of the rational basis test
o FOOTNOTE 4

Court will generally show deference to legislature, but deference will not extend to laws
that:

Interfere with individual rights

Restrict ability of the political process to repeal undesirable legislation

Discriminates a discrete and insular minority

Provided framework for judicial review

If the right is fundamental = strict scrutiny

If the right is not fundamental = rational basis


Economic Substantive Due Process Since 1937

Since Carolene Products, SCOTUS has not overturned a single piece of economic regulation on the
basis of substantive due process

This is because economic substantive due process is subject to rational basis scrutiny

26

26

Two Main Types of Substantive Due Process

Economic Substantive Due Process

Personal Liberties Substantive Due Process


Economic Substantive Due Process is subjected to Rational Basis Scrutiny

Carolene Products
o Rule

Laws regulating business and employment practices will be upheld when challenged
under the due process clause so long as they are rationally related to serve a
legitimate government purpose.

The result is that, after 1937, SCOTUS has never found an economic regulation to violate substantive
due process
The Main Tests of Substantive Due Process and Equal Protection

Strict Scrutiny
o Test: Government must show that the challenged classification serves a legitimate compelling
state interest and that the law is narrowly tailored to serve that interest.
o Restrictions resulting in Strict Scrutiny Analysis

Anything that restricts a fundamental right

Examples: Denial/Dilution of right to vote; Interstate migration; Access to courts; other


rights recognized as fundamental
o Suspect Classifications resulting in Strict Scrutiny

Race; Religion; National Origin; Alienage


o Notes

27

27

Plaintiff almost always wins


Intermediate Scrutiny
o Test: Government must show that the challenged classification serves a legitimate important
state interest and that the law is substantially related to serving that interest
o Quasi-Suspect Classifications result in Intermediate Scrutiny

Gender; Non-marital children


o Notes

This is primarily applicable to equal protection cases


Rational Basis
o Challenger must show that the challenged law is not rationally related to serving a
legitimate government interest
o Notes

Government almost always win

If it doesnt meet strict scrutiny or intermediate scrutiny qualifications, it gets rational


basis scrutiny

Inclusiveness

This question goes to the issue of whether a classification is narrowly tailored or substantially related.

Over-Inclusive A law is over-inclusive if it applies to some people who need not be included for the
government to achieve its legitimate purpose

Under-Inclusive A law is under-inclusive if it does not apply to some people who should be
included for the government to achieve its legitimate purpose.

Most statutes are over- or under-inclusive to some degree.


o Strict Scrutiny: Very harsh on over-inclusive statutes that create restrictions
o Rational Basis: Courts are very lax on the relationship between interest and inclusivity
What makes a legitimate state purpose? (Rational Basis)

The traditional police powers of the state make a good example


o States may take action for the purposes of public health, safety, or morals

Almost any goal not prohibited by the Constitution can be a legitimate state interest

28

28

Substantive Due Process Analysis

Is there a liberty or property interest at stake?


o If yes, proceed.
o If no, dismiss entirely.

Is there a fundamental right at stake?


o If yes, strict scrutiny
o If no, rational basis
Is the right infringed?
o When exercise of right is prohibited, there is no doubt about infringement
o Burdening the right

Court considers the directness and substantiality of the interference

Example: Government infringes upon a right when it requires the citizen to give up a
constitutional right in order to receive a government benefit.
Is the government action justified by sufficient purpose?
o Strict scrutiny requires a compelling government interest

Government has the burden of showing that a truly vital interest is served by the law in
question
o Rational basis merely requires a legitimate purpose
Are the means sufficient related to the goal sought?
o Rational Basis

Merely means the government must act in a manner that is reasonably related to
achieving the goal

The government is not required to use the least restrictive means


o Strict Scrutiny

Government must show that the law is necessary to achieve the objective, and;
government could not obtain the goal with a means that was less restrictive of the right

What Constitutes a Fundamental Right?

Michael H and Glucksberg


o Analysis

29

29

What is the right? How narrow is the right?


Look at the history and heritage to determine whether it actually was a fundamental
right.
Today, the Court has moved toward a strict version of this test, often holding that history and
heritage are how we determine fundamental rights.
How we frame the right often determines whether it is a fundamental right.

Example Michael H: Is it the right of parenthood, or the right of a father to have a


relationship with biological daughter
Scalia has said that the Court should use the historical tradition most specifically related to the
issue in order to determine the existence of a fundamental right

o
o

Note
o

The Court has not found the existence of a new fundamental right since Roe v. Wade, and is
not likely to do so anytime soon.

Right to Privacy

In General
o Griswold v. Connecticut

Declared a law prohibiting the distribution of contraceptives to be unconstitutional

Privacy became a fundamental right

Found privacy to be implicit throughout the Bill of Rights in the penumbras of


the different amendments

The penumbral approach has long-since been abandoned


o Eisenstadt v. Baird

Law prohibiting distribution of contraceptives to unmarried persons declared


unconstitutionall

Expands Griswold determination that control of reproduction is a fundamental right

Privacy as a fundamental right: If the right of privacy means anything it is the right
of the individual, married or single, to be free from unwarranted governmental
intrusion into matters so fundamentally affecting a person as the decision whether to
bear or beget a child.
o Whalen v. Roe

Privacy involve two kinds of interests

Individual interest in avoiding disclosure of personal matters

Interest in independence in making certain kinds of personal decisions

The Abortion Cases

30

30

Other
o

Roe v. Wade

Held that women have a fundamental right to make personal decisions about
whether or not to terminate a pregnancy.

Strict Scrutiny

Created the trimester system determining when state may regulate abortion.
Planned Parenthood v. Casey

Held

SCOTUS decides what rights are incorporated into the 14th Amendment

Roe should not be overruled

Abolishes the trimester system

Creates the Undue Burden Test

Does a state regulation of abortion have the purpose or effect of


placing a substantial obstacle in the path of a woman seek an abortion
of a nonviable fetus?

What level of scrutiny was used?

This was not strict scrutiny, but its not clear what it is

If not strict scrutiny, was it still a fundamental right?


Gonzalez v. Carhart

Held

Applied Undue Burden Test, but it sounded like rational basis

Congress had a rational basis to act in refusing to allow partial birth abortions
because there is uncertainty about medical necessity

Prof: There were a lot of ticking time bombs in this case, placing the right to an
abortion in a very precarious situation
Cases
Glucksberg v. Washington (right to die case)

Lesson: Failure to demonstrate a fundamental right will result in rational basis being
applied

Held: History and heritage show that right to die is not a fundamental right
Michael H (parental rights case)

Lesson: How we frame the right is often determinative of whether it is a fundamental


right.

Obergefell Gay marriage


SCOTUS found a fundamental right
Did SCOTUS apply strict scrutiny
o
Found a fundamental right, but did not talk about strict scrutiny - which would
have meant the court would have had to talk about a compelling state interest (to
restrict marriage to a man and woman)
o
Suspect classification - gets strict scrutiny
o
Quasi = heightened scrutiny/ intermediate scrutiny
Quasi would be gender/illegitimate children

Court looked at the history of marriage

Greater Caution - not in this case,

Only answered the right to marry - did not address sexual discrimination
There is collateral from the decision, SCOTUS did not define this - just the marriage

issue
o
Could have been different if court would have found "suspect class" or "quasi
class" then would have given some answers. Would have gone a long way to later
equal protection cases

31

31

Windsor (DOMA Case)


o Issues: Standing (prudential looks like a collusive suit); Substantive Due Process
o Held:

Standing: Much like Craig v. Boren, case could proceed because the third-party
interveners argued the case well

Substantive Due Process: Court never said what level of scrutiny was applied. Just said
that a liberty interest was violated because the statute demeaned same sex couples.

Basic Explanation

Equal protection requires the government to treat similarly situated people in similar ways

People who are dissimilarly situated need not be treated in similar ways

The question is not whether there is discrimination the question is whether the government has
a sufficient basis for the discrimination

The equal protection clause permits classifications (discrimination) for constitutionally


sufficient reasons
History

Equal protection was given no weight until Brown v. Board of Education

Today, equal protection is a first resort, rather than a last resort

Reverse incorporation 5th Amendment incorporates the 14th Amendments equal protection clause
against the Federal Government
Basics

Equal protection cases test the validity of the classification, not whether the claimant belongs to
one group or another group
o To which group the claimant belongs is a question of fact, not of law.

The Tests
o Strict Scrutiny
o Intermediate Scrutiny
o Rational Basis

Substantive Due Process vs. Equal Protection


o SDP

A law that burdens the ability of all persons to exercise a fundamental right will be
examined under SDP

Is the restriction justified by a sufficiently legitimate government interest?


o Equal Protection

A law restriction of rights that places burdens on one classification of persons

Is the classification justified by a sufficiently legitimate government interest?

The Claim
o Equal protection merely challenges the classification itself. If the challenge succeeds, the
legislature must decide to either apply the restriction to everybody or nobody (cannot restrict
based on the classification).

32

32

Analyzing an Equal Protection Case

Step One

33

33

Is there a classification? If so, what is the classification?

Step Two
o Does the classification distinguish among people who are similarly situated? (suspect?)
Step Three
o What is the appropriate level of scrutiny?
Step Four
o Does the government purpose meet the scrutiny tests requirements?

Three Types of Classifications (Racial and Gender Discrimination)

Facial Classifications
o The statute on its face creates a classification
o Example: No woman may serve as a police officer

Facial neutrality, but with discriminatory impact


o Statute is neutral in terms of language, but the impact of the language creates a discriminatory
effect
o Example: All police officers must be at least 6 tall and weigh at least 160 pounds
o In order to succeed, complainant must show a discriminatory impact and a
discriminatory purpose (improper purpose in gender discrimination)

Example: Jim Crow laws Grandfather voting clauses

Facial Neutrality, but with discriminatory application


o Statute is neutral in terms of language, but the statutes application is discriminatory
o Example: Yick Wo v. Hopkins Asian petitioners for laundry licenses were continuously denied
for 20 years, while non-Asians had their licenses granted
o This is the hardest type of equal protection case to win. Usually needs statistical proof.
o In order to succeed, complainant must show discriminatory impact and intent by the
decision maker to discriminate
The Main Tests of Substantive Due Process and Equal Protection

Strict Scrutiny
o Test: Government must show that the challenged classification serves a legitimate compelling
state interest and that the law is narrowly tailored to serve that interest.

34

34

Restrictions resulting in Strict Scrutiny Analysis

Anything that restricts a fundamental right

Examples: Denial/Dilution of right to vote; Interstate migration; Access to courts; other


rights recognized as fundamental
o Suspect Classifications resulting in Strict Scrutiny

Race; Religion; National Origin; Alienage


o Notes

Plaintiff almost always wins


Intermediate Scrutiny
o Test: Government must show that the challenged classification serves a legitimate important
state interest and that the law is substantially related to serving that interest
o Quasi-Suspect Classifications result in Intermediate Scrutiny

Gender; Non-marital children


o Notes

This is primarily applicable to equal protection cases


Rational Basis
o Challenger must show that the challenged law is not rationally related to serving a
legitimate government interest
o Notes

Government almost always win

If it doesnt meet strict scrutiny or intermediate scrutiny qualifications, it gets rational


basis scrutiny
o

Inclusiveness

This question goes to the issue of whether a classification is narrowly tailored or substantially related.

Over-Inclusive A law is over-inclusive if it applies to some people who need not be included for the
government to achieve its legitimate purpose

Under-Inclusive A law is under-inclusive if it does not apply to some people who should be
included for the government to achieve its legitimate purpose.

Most statutes are over- or under-inclusive to some degree.


o Strict Scrutiny: Very harsh on over-inclusive statutes that create restrictions
o Rational Basis: Courts are very lax on the relationship between interest and inclusivity

35

35

What makes a legitimate state purpose? (Rational Basis)

The traditional police powers of the state make a good example


o States may take action for the purposes of public health, safety, or morals

Almost any goal not prohibited by the Constitution can be a legitimate state interest
Economic Regulations that establish Classifications

These get rational basis review

Remember, government doesnt have to be right, it just has to be rational

There must be some relationship between the law and the states purpose
Types of Discrimination

Invidious Discrimination Disadvantages minority group

Benign Discrimination Benefits minority group


Suspect Classifications

These classifications trigger strict scrutiny

Race; Religion; National Origin; Alienage

Racial Classifications
o Must determine the type of classification (Facial, Facially-Neutral), and type of discrimination
(Invidious or Benign)
o All racial classification get strict scrutiny, whether they are benign or invidious
(Adarand v. Pena)

There may be a compelling state interest to rectify past discrimination in some cases
(such as employment), but not in others

Alienage or National Origin Discrimination


o Alienage: Discrimination based on status as a non-US citizen
o National Origin: Discrimination based on ancestry or ethnicity
o Generally treated no differently than racial classifications

36

36

Exceptions

Alienage

Right to vote, hold office, or serve on juries

Rational basis

Immigration Laws

Congress discriminates against these groups with immigration laws, and those
classifications are given rational basis scrutiny

Gender Classifications

Given Intermediate Scrutiny

Must determine the type of classification (Facial, Facially-Neutral), and type of discrimination (Invidious
or Benign)

Holdings on Gender Classifications


o Classifications are based on gender stereotypes Unconstitutional (Hogan)
o Classifications benefitting women to remedy past discrimination Permitted
o Important state purpose creates classification based on genuine biological differences between
the sexes (Unwed mothers have presumptive custody over unwed fathers)
Non-Marital Children

Given Intermediate Scrutiny

These cases rarely occur

Given this level of scrutiny because this was one a huge issue, but states have moved away from these
laws (exception: intestacy laws still differentiate to avoid fraud)
Other Classifications (Rational basis)

Quasi-Suspect Classification
o Cleburn Some people wanted to build homes for the mentally disabled to live in. Statute
prohibited mentally disabled from living in group homes without a permit. Held invalid under
rational basis (with a bite as the dissent referred to it)

Wealth Classifications
o Given rational basis scrutiny
o Claim is made that discrimination based on financial circumstances should get a suspect
classification SCOTUS rejected

It is within discretion of decision maker as to who gets benefits

Age Classifications
o Most age discrimination claims are made under statutory or regulatory law

Those provide much tougher standard to uphold the discrimination

Constitution, however, merely requires rational basis

If covered by a statute or regulation, do not bring age discrimination claim under equal
protection

37

37

Fundamental Rights

Voting rights

Right to Travel and Interstate Migration

Remember Fundamental rights get strict scrutiny analysis


o Compelling state interest + narrowly tailored
Voting Rights

Introduction
o 15th Amendment Right of citizens to vote shall note be abridged on account of race, color,
or previous condition of servitude.
o 19th Amendment Womens suffrage
o 24th Amendment Outlawed poll taxes
o 26th Amendment Changed voting age to 18 for federal and state elections

Durational Residency Requirements


o The requirements demand one to live within a state for a certain period of time before being
allowed to register to vote

Purpose: Prevent voter fraud with people coming in from out of state
o Held: Preventing voter fraud is a compelling state interest, but duration is an issue

1 year: Too long and too over-inclusive

50 days: Upheld as sufficiently narrowly tailored

38

38

Special Interest Elections


o Hill v. Stone

Can state/locality restrict who can vote in special interest elections?

Held: No. Whether someone pays taxes on that interest or not, they have a general
interest in the community therefore, the community can vote.
Felony Disenfranchisement
o Richardson v. Ramirez Constitutionally permissible to disenfranchise those convicted of a
felony (14th Amendment, 2)
o Critics say this is racially skewed, but there is no equal protection claim to be made
First Amendment Protections for Voting
o 1st Amend.: Congress shall make not law abridging the right of the people to peaceably
assemble, and to petition the government for a redress of grievances.
o Primary Elections cases decided under right to assemble

States requiring people to register with a political party is completely constitutional

Compelling interest: Prevent election tampering

Alternate System: Everyone permitted to vote in primaries

Also constitutional

Compelling interest: Protect right to privacy no one has right to know who
youre voting for

o Issues:

Kusper v. Pontikes SCOTUS struck down law prohibiting a person from voting in a
primary election for one party on the basis that they voted in another partys primary in
the last 23 months
Access to the Ballot
o There is no constitutional right to run for election, but restriction of candidates infringes on
right to vote
o There are compelling state interests in restricting access to the ballot

Limit number of people on ballot

Avoid voter confusion

Preserve integrity of voting process

Produce winners with strong majority/plurality


o How do you constitutionally limit the ballot under strict scrutiny?

39

39

Impermissible

High filing fees Not narrowly tailored (Bullock & Lubin)


Permissible

Petition System Require a certain number of signatures to get onto the ballot

Major parties dont have to participate If a party gets enough votes in


previous election, they are automatically on the ballot the next election

Right to Travel / Interstate Migration

These rights do not pertain to intra-state travel or international travel


o If family moves within same state, or someone wants to travel to another country, the
fundamental right to travel does not apply

Example issue: A family moves from one state to another, and is denied benefits under a statute
designed to impair interstate travel unconstitutional

Difference between durational requirement and bona fide residency requirement


o Durational requirements have a waiting period after establishing residency
o Bona fide residency requirements provide the benefit as soon as residency is established

Bona fide residency requirements Rational basis

Durational residency requirements Strict scrutiny IF classifying between citizens

Shapiro v. Thompson Durational residency requirements get strict scrutiny


o The requirements are constitutionally permissible, but they must meet SS requirements
o Delaying assistance benefits an entire year was impermissible

Shorter durations have been upheld

McCarthy v. Philadelphia Firefighter fired from job of 16 years because he moved to New Jersey
o This was a bona fide residency requirement
o This gets rational basis

Martinez v. Bynum Does equal protection permit school districts to require students to be residents
of the school district?
o Yes. Bona fide residency requirement As soon as kid becomes resident, he can attend

Saenz v. Roe (Privileges and Immunities Clause case)


o California allowed new residents to get welfare benefits immediately, but they were limited
based on previous states benefits
o Held: Privileges and Immunities Clause applies to interstate travel

Rights regarding travel

Right to enter and leave a state

Right to be treated as a welcome visitor when present temporarily

Once residency is established, right to be treated like other state citizens

Did not overrule Shapiro

1. Today, discrimination cases are decided primarily based on statutes, rather than the
Constitution

(Civil Rights Act, Anti-Discrimination Statutes, ADA, Age Discrimination Act, etc)

Cases interpreting these statutes and applying them are not engaging in equal protection analysis
2. Dont forget state laws

Remember, the adequate and independent state grounds doctrine may apply

States can provide greater protections for their citizens than those provided by the federal constitution
3. Some claims that might be brought under equal protection may be decided under collective
bargaining rights.

Collective bargaining agreements can determine someones rights in an employment scenario

Contractual rights instead of constitutional rights


4. Equal protection may provide relief where a specialized statute does not

Ex: Title IX only applies to schools receiving federal funding. Would have to bring equal protection case
if school is not subject to Title IX.

40

40

Difference between substantive due process and procedural due process

Substantive due process challenges the final government decision

Procedural due process claims challenge the procedures the government used in reaching its
decision
o Normally, this means:

Notice

41

41

An opportunity to be heard (i.e., a hearing)


And an impartial decision maker

Analyzing a Due Process Claim

1. Has there been a deprivation;

2. Of life, liberty, or property;

3. Without due process of law?


Note: We dont generally care about life in the context, because those cases rarely come up. Were
primarily talking about liberty and property interests
What Constitutes a Property Interest?

Board of Regents v. Roth


o To have a property interest in a benefit, a person must have more than an abstract need or
desire for it. He must have more than a unilateral expectation of it. He must, instead, have a
legitimate claim of entitlement.
o The legitimate claim of entitlement comes from the law or some understanding or relationship
with the state.

Test
o Plaintiff must have a legitimate claim of entitlement to the property interest
o The legitimate claim of entitlement must come from the law or some understanding or
relationship with the state.
What are property interests?

Property rights may be tangible or intangible

Tangible
o Real and personal property

Intangible
o Employment rights (but not at-will employment)
o Tax refunds
o Social security benefits

42

42

o
o

State-issued licenses (once issued, the license becomes a property right)

Drivers, occupational, liquor, etc


A lot of other stuff

Property Cases

Arnett v. Kennedy Bitter with the Sweet Doctrine: If you take the sweet, you also take the bitter
(take job, you get whatever the employment contract gives you)

Bishop States can make employment at-will (thereby eliminating employment rights)

Cleveland v. Loudermill Once there is a property right, how much process is due is an issue of law.
What Constitutes a Liberty Interest?

Types of Liberty Interests


o Freedom from government-imposed physical restraint

(e.g. criminal prosecutions, involuntary civil commitments includes mental


institutions, foster care and juvenile detention)
o Freedom from government-imposed restraint on the free exercise of fundamental rights

(Such as textual constitutional rights and privacy)


o Freedom from other government-imposed restraints on choice or action

(e.g. personal reputation)


Reputation as a Liberty Interest

Paul v. Davis
o If only claim to harm under procedural due process is loss of reputation, you must show some
tangible or financial loss in order to have a due process claim.
Deprivation of Due Process

In order for an act to violate ones right to procedural due process, there must be a deprivation

This issue is heavily litigated in three areas


o Government negligence

43

43

Government negligence does not constitute a deprivation (Daniels)


Availability of State Remedies

If there is a state remedy available, such as a tort remedy, then there is likely no valid
procedural due process claim

Parratt and Hudson Ability to invoke state tort remedy means no PDP violation

Note:

State tort remedies only trump PDP if the action by the government was an
isolated, individual act

If the action is part of the governments policy or normal behavior within the
agency, then there is a 14th Amendment procedural due process claim

It is very difficult to show that a state has a policy that violates PDP
Failure to Protect while in Custody or Confinement

Those who are in state custody and cannot leave are entitled to a minimum level of
protection

Deshaney Facts: Failure by agency to take child into custody, he then gets hurt.
Held: No deprivation of a liberty interest when violence is committed against someone
who is not in state custody

Castle Rock v. Gonzalez Facts: Father violated protective order, and police did not
respond to 911 calls. Mother tried to claim a property entitlement to enforcement of
protective order. Held: No. There is only a property right if there is a legal entitlement.
If there is discretion, there is no entitlement.

What Constitutes Due Process of Law?

General constitutional standard: You get minimal procedure before the decision is made, and you
have an opportunity for a fuller hearing after the decision is made

Typical Procedural Due Process afforded to Individuals


o Informal pre-decision review or conference
o Fuller post-decision hearing
o Judicial review (if requested)

Test (Mathews v. Eldridge)


o Consider:

The private interest that the government action will affect

The risk of erroneous deprivation through the procedures used, and the probable value,
if any, of additional safeguards

The governments interest, including fiscal and administrative burdens

The Test (explained)


o Private interest

44

44

The interest of the claimant


Does the law consider the interest more or less important? (objective test)
Examples of important interests: Public education, occupational licenses, public
assistance benefits
o Risk of erroneous deprivation

Would more procedure reduce risk of an erroneous decision?

If issue turns on law, less reason to have a full hearing

If issue turns on facts and will require testimony, more reason to have full hearing
o Fiscal and Administrative Burdens

Guaranteeing full procedural due process is important. Cost is considered in


determining how much process someone is entitled to.
Administrative Agencies have two types of authority
o Rulemaking authority

So long as they promulgate rules according to proper procedures, they meet the
procedural requirements
o Adjudicatory authority

They have the authority to adjudicate cases and act like courts
General Framework
o Before enforcement of decision

Very little procedural due process

Usually a small, informal hearing


o After enforcement

Entitled to a more fleshed out procedure


A person is always entitled to at least the amount of procedure provided by the agencys rules and
regulations
Can an agency just ignore content of hearing and make a decision?
o While courts provide a lot of deference to administrative agencies, courts will overrule their
decisions if they were acting irrationally.
o

Who is subject to the constitution?

45

45

State Action Doctrine


o Nearly every constitutional provision applies ONLY to the government

(federal, state, municipal, agencies, agents, etc)


o If state action is found, there is normally an avenue for recovery
o It is rare for the state action doctrine to be at issue
Exceptions

Government Function
o Private entity performs a function normally performed by the government

Entanglement
o Private entity works very closely with the government as it does something
Government Function Exception

If the private person or entity is performing a function that is normally or exclusively done by the
government, then the private person is a state actor and can be treated that way for constitutional
purposes

Most law comes from the White Primary Cases


o Facts: Statutes prevents blacks from voting in democratic party primaries
o Held: Running elections is a government function, even when they are only primaries,
therefore these restrictions violated the constitution

Other cases
o Marsh v. Alabama Company towns (entire towns created by corporations for their
employees) subject to constitution, because the running and maintenance of a town are things
normally done by government
o Evans v. Newton Running of parks left to government. A private trustee for a park is subject
to constitution.

Shopping Mall Cases


o Logan Valley Plaza held that labor picketers had a right to access a private shopping mall,
because mall functions as a town center
o Hudgens v. NLRB Overruled Logan

Flagg Brothers
o Facts: Sheriff sent family to a storage company to store their belongings after eviction. Storage
company lost property.
o Held: Government function exception only applies to those things exclusively performed by the
government.
o Note: Court has moved to a very narrow interpretation of this exception, even though many
things have been privatized

46

46

Government Entanglement

Activity by a private entity becomes state action when the private entity is so entangled with the
government that private conduct becomes government activity.

Cases
o Burton Coffee shop had lease from government in public parking garage. Held there was
entanglement
o Rendell-Baker Private school with 99% of funding from state. Held no entanglement
o Moose Lodge Private club had liquor license from the state, and was regulated by the state.
Held no entanglement, even though there was a stronger connection than in Burton.
o Brentwood I Privately run athletic association acts as organization for high school sports.
Held that there was entanglement. Remanded for determinations of constitutional violations.
o Brentwood II No violation. High school voluntarily joined association, must abide by its rules.

These cases show how unclear the entanglement exception really is

The Basics

Speech means more than just verbalization it means expression

Weighing process is weighted in favor of protecting speech

Most (not all) 1st Amendment speech claims get strict scrutiny

The right is not absolute

1st Amendment holds a preferred position in American law


Preferred Position

Narrow presumption of constitutionality


o Normally, the presumption is that the statute is constitutional
o But when dealing with speech, that presumption is practically reversed

Especially if the restriction is content-based

Strict construction to avoid limits on expression


o In speech cases, legislation is interpreted strictly based on its language

Relaxed rules of standing


o Courts more likely to allow third-party standing
o Overbreadth Doctrine

If litigant can demonstrate Overbreadth, then the court will strike down the statute,
even if it can be applied to the litigant in a constitutional manner

Near-absolute ban on prior restraints


o What is a prior restraint?

It is the government action to forbid someone from speaking before they say anything

Normally, government can get an injunction preventing some action. But in speech
cases, it is almost impossible for the government to get that injunction.

Higher standards of procedural due process


o Example: Parade permits
o Permits can be a form of prior restraint
o If permits are content neutral restrictions, then they get rational basis scrutiny.
Challenging a Statutes as Violative of the 1st Amendment

Facial Challenges
o Total invalidation of the law Claimant asserts that the law cannot validly be applied to punish
anyone.
o Even if claimants speech could be regulated under the 1st Amendment, if the statute is
unconstitutionally vague or overbroad, it will be struck down as a facial violation

As-Applied Challenge
o Claimant says that the law may be constitutional on its face, but he cannot be convicted for his
actions

47

47

Example: Statute prohibits obscenity (facially valid). Claimant says that his conduct does not
constitute obscenity, and therefore he cannot be punished for it.

o
Vagueness and Overbreadth in Speech

Void for Vagueness


o Test

The statute must define the criminal offense with sufficient definiteness so that:

48

48

Ordinary people can understand what conduct is prohibited, and

The statute does not encourage arbitrary and discriminatory enforcement


o SCOTUS says second prong usually more significant
o In 1st Amendment cases, if a statute is struck down for vagueness, it is completely struck
down
Overbreadth
o Test

A law regulating expression is unconstitutionally overbroad if it reaches not only


unprotected expression, but also a substantial amount of protected expression.

There must be substantial Overbreadth (Broadrick v. Oklahoma)


o If a statute is struck down as overbroad, it is completely struck down
Reasons for these doctrines
o Prevent chilling effects
o Prevent selective enforcement of speech laws
o Create incentive to draft good legislation
o Judicial modesty give legislature another chance to get it right
These doctrines are not the same thing
o While it is true that many statutes meet the requirements of both if they meet one, it is
possible for a statute to be overbroad without being vague.

Distinction between Speech and Conduct

A conduct statute may be struck down as to the defendant, but a speech statute will be struck down
entirely

Conduct statutes get rational basis

Speech statutes get strict scrutiny


Prior Restraints and Permit Systems

Prior restraint occurs when the government prohibits or prevents someone from speaking before they
speak.

Typical Prior restraints


o Administrative systems requiring a license or permit before a parade or demonstration

49

49

Routinely upheld so long as they are content-neutral


o Injunction that enjoins person from engaging in specific future expression
o Other government acts, such as prohibiting publication of information in a newspaper
Generally
o Almost impossible to save a content-based prior restraint
o Content neutral prior restraints are normally upheld
Content-Based Prior Restraints
o Content-Based restrictions

Distinguish between expression based on what the speaker says, or what the speaker
wants to say.

Example: Pentagon papers cases

Presumptively invalid upheld only where regulation is narrowly tailored to achieve


a compelling interest)

Short of publishing treasonous information, it is almost impossible for these to


be upheld
o Content-Neutral restrictions

Operates regardless of the expressions viewpoint or subject-matter

Requires viewpoint and subject-matter neutrality

Viewpoint neutral Government does not regulate expression based on messages


position (i.e. does not differentiate between pro-life or pro-choice)

Subject-matter neutral Government does not regulate expression based on the


expressions topic (i.e. Government cannot ban all picketing in a neighborhood,
except picketing in connection with a place of employment)

Intermediate scrutiny Valid if narrowly tailored to achieve an important


government interest, and they must leave open ample alternative channels of
communication
Permit Systems
o The power of the permit

The authority of local governments to determine the conditions under which a private
user may use public property (time, sound volume, must clean up, etc)
o Test

Content-neutral license or permit systems are permissible when:

It has an important reason that provides ample alternatives

Public safety is generally an important reason

It establishes strict criteria for grant or denial, leaving almost no contentbased discretion; and

50

50

It guarantees procedural safeguards, such as prompt determination of


license applications, and an opportunity for a hearing before expression is
restrained

Time, Place, and Manner Restrictions

Expression and speech need a place to occur


o Public property often becomes that forum

The Public Forum Doctrine


o People do not have the right to protest on private property if the owner does not want them
there.
o Peaceable assembly refers to public property.

Types of Forums
o Public Forum

Public property that the government must make available for expression

Examples: Public streets, sidewalks, and parks

Schneider v. New Jersey Cities must shoulder the burden of costs of demonstrations
in public areas. Cannot refuse to allow demonstrations on the basis of cost.
o Limited (Designated) Forum

Public property that could be closed to expression entirely, but the government has
chosen to allow it to remain open

Examples: Meeting rooms in city hall and libraries; classrooms in public schools

Note: once a city opens a limited forum up to someone, they have to open it to
everyone on a non-discriminatory basis

If they want area to remain closed, they should have a policy indicating that,
and then enforce the policy
o Non-Public Forum

Public property that the government may close to all expression, or where the
government may restrict expression, provided the restriction is reasonable and
content-neutral

Examples: Operating rooms in hospitals, military bases, post office property

Public Forum Analysis


o Content-based restrictions are presumptively invalid (strict scrutiny)

Upheld only where the government provides a regulation that is narrowly tailored to
accomplish a compelling interest
o Content-neutral restrictions are valid IF:

They serve a significant government interest

Public order/safety; aesthetics (ban billboards on highways); tranquility/privacy

They are narrowly tailored

They leave open ample alternative channels

51

51

Commercial Speech

Test (Central Hudson Gas & Elec. V. Public Service Commission)


o Is the restriction justified by a substantial government interest?

Note: Easier to uphold restriction on commercial speech than regular speech


o Is the advertising false or deceptive, or does it concern illegal activities?
o Does the law directly and materially advance the substantial government interest?
o Is the regulation no more extensive than necessary to achieve the substantial government
interest? (i.e., is it substantially related to that interest?)

Commercial speech essentially gets intermediate scrutiny

Substantial government interest In commercial speech, focus is on recipient of the speech, and
how it effects them
o Example: Prohibition on advertising for cigarettes near schools

False or Deceptive Not a very important factor; cure for false speech is more speech (marketplace
of ideas)
o SCOTUS has never used this prong to strike down commercial speech

Materially advance Not a very important factor government acting against speech implies that
the act advances the governments interests

Substantially related If regulation causes a greater prohibition on commercial speech than is


necessary, or it would create too great of a burden, it is not substantially related
o This factor killed the regulation in the cigarette case impossible to advertise cigarettes in
urban areas because there are so many schools

What Constitutes Commercial Speech?


o Commercial speech is speech that does no more than propose a commercial transaction.
Advertisers interest is purely economical.
o Three characteristics

Advertisement of some form;

Refers to a specific product;

The speaker has an economic motivation

52

52

Substantial Interests at play in Commercial Speech


o Advertisers interest in disseminating the message
o Consumers interest in receiving the message
o Governments interest in restricting the message

Symbolic Speech

The name given to a doctrine that determined when conduct is expressive (conduct is protected as
speech)

Questions
o When is conduct expressive?
o When may government regulate expressive conduct?

When is conduct expressive?


o Test (Spence v. Washington)

Did the person intend to convey a particularized message?

In the surrounding circumstances, would the message be understood by those who


viewed it?

When may the government regulate expressive conduct?


o Test (OBrien)

The government regulation must further an important or substantial government


interest

The government interest must be unrelated to the suppression of free expression

The restriction must be no greater than is necessary to further the government


interest.

City of Erie v. Paps


o Demonstrates how far Court will go to find expressive conduct

Strip club owner claims violation of his 1st Amendment rights after ordinance is passed
to eliminate strip clubs

Court said this strip club was on the outer periphery of expressive conduct

City needed a substantial interest in shutting down the strip club

Held: Restriction is constitutional. Passed the OBrien test

After this, if there is something that seems like a close case for expressive conduct, it
seems like the Court would hold that there is expressive conduct (if theyre willing to
protect strip clubs)

53

53

Types of Unprotected Speech

Incitement to Unlawful Activity (Brandenburg)

Obscenity

Child Pornography

Fighting Words

True Threats
Restricting dangerous speech (Incitements to Unlawful Activity)

The History
o Clear and present danger test reasonableness test risk formula Brandenburg

Brandenburg Test: Intent + Imminence


o The speech must be directed to inciting or producing imminent, unlawful action AND
o The speech must be likely to incite or produce imminent, unlawful action
o This test is very protective of speech
Obscenity

Difference between obscenity and pornography


o Obscenity is the worst of the worst
o Pornography (except child porn) is protected by 1st Amendment
o Obscenity is not

Justice Stewart: I know it when I see it (Jacobellis v. Ohio)

In order to determine whether something is obscenity, the judges actually have to view it.
o That can be awkward

Miller Test (Miller v. California)


o Would the average person, applying contemporary community standards, find that the work,
taken as a whole, applies to the prurient (lustful) interest?

54

54

o
o

Contemporary community standards -> subjective (location matters)


Does the work depict or describe, in a patently offensive way, sexual conduct specifically
defined by applicable state law?
Does the work, taken as a whole, lack redeeming artistic, literary, political, or scientific value?

Child Pornography

Regular pornography is protected, child pornography is not protected (New York v. Ferber)
o Child porn does not qualify as obscenity, but it remains unprotected

Defined
o Non-obscene photograph and films depicting sexual activity by juveniles.

Virtual Child Pornography


o Ashcroft v. Free Speech Coalition

Pornography involving non-existent children is protected by the first amendment

This holding required prosecutor to prove something they couldnt prove (that child was
real)
o Congress Response

Criminalized the offer, distribution, or acceptance of child pornography (real or virtual)


o US v. Williams

Offer or receipt of child pornography is conduct, not speech

Therefore, the statute is valid, even as to virtual child pornography


Fighting Words

Fighting words are outside of the protection of the 1st Amendment


o Fighting words have no redeeming social value
o Not an essential part of any exposition of ideas

Defined
o Fighting words are those by which their very utterance inflict injury or tend to incite an
immediate breach of the peace, and are not constitutionally protected.

Chaplinsky Holds fighting words are unprotected


o Court has not upheld a fighting words conviction since Chaplinsky. It is basically a dead
doctrine, but it is still good law.

Wisconsin v. Mitchell A physical assault is not expressive conduct protected by the 1st Amendment
True Threats

Defined
o Encompass those statements where the speaker means to communicate a serious expression
of an intent to comit an act of unlawful violence to a particular individual or group of
individuals.
o The speaker need not actually intend to carry out the threat.
o A prohibition on true threats protects individuals from the fear of violence and from the
disruption fear engenders.

Test
o Intimidation is the constitutionally proscribable sense is a type of true threat, where the
speaker directs a threat to a person or group of persons with the intent of placing the victim
in fear of bodily harm or death.

Wisconsin v. Mitchell A physical assault is not expressive conduct protected by the 1st Amendment

55

55

The First Amendment does not explicitly grant the Freedom of Association
What is the Freedom of Association?

It is the freedom to join as a group with other like-minded individuals

NAACP v. Alabama State passed a statute requiring publication of membership lists of foreign
corporations in order to intimidate people and keep them from joining NAACP. Held: There is a
freedom to associate. Reasoning: There is strength in unity. For freedom of speech to have meaning,
you must be able to associate with others.

NAACP v. Caliborne Held: A peaceful boycott is a 1st Amendment protected activity.


Freedom of Association and Discrimination

What happens when someone files a lawsuit against a club that discriminates on the basis of race,
gender, religion, etc.?
o 1st Amendment Freedom of Association may overcome discrimination statutes in some
circumstances

Two circumstances were groups may discriminate


o Intimate Association

A small, exclusive group had the 1st Amendment right to exclude

Example: A bridge game of local women

Once you go beyond a handful of people, it ceases to be an intimate association


o Expressive Association

A group has a 1st Amendment right to exclude so members can associate to express
their views on political, economic, cultural, or social affairs

Examples:

College Republican group can require members to actually be members of the


Republican Party

KKK can reject blacks, jews, and catholics

Rationale: Refusing to allow these groups to exclude would compromise the groups
entire purpose by requiring them to allow people to join who have positions
antagonistic to the group.

If it can be shown that a group has an expressive purpose, and having a nondiscriminatory membership policy would jeopardize the groups purpose, then a group
is permitted to discriminate in its membership.
o Cases

Rotary Club Requirement that group admit women to local rotary clubs did not deny
freedom of association or expressive association

Boy Scouts of America v. Dale Boy Scouts have anti-gay views, and may deny
membership to gays on the basis of expressive association.

Citizens United (the political contribution case) Held that corporations and labor
unions can spend unlimited amounts of money in a political campaign. Rationale:
People can associate with one another in the act of spending money to assist a political
position.

56

56

Article I, 8 contains the most complete enumeration of congressional power


Congressional Power

The states have the police power (an implied power)


o The police power is the power to do anything that the federal constitution does not prohibit
(permits states to take action for public health, safety, and morals)

Congress only has enumerated powers (no police power)


o Congress must have the power to act granted to it by the Constitution
o Its power is limited by the powers enumerated in the Constitution
McCulloch v. Maryland

Considered the most important SCOTUS case of all time

Stands for: If Congress has express authority, that [authority] carries with it the implied authority to
carry the express authority provisions into being by any means not inconsistent with other provisions of
the Constitution.
o Explanation: Express grants of power to Congress are accompanied by implied grants of power
to effectuate is express authority

Also: Necessary and proper clause simply means convenient rather than actually meaning
necessary
o Debate: Jefferson said it mean indispensible, while Hamilton believed it mean
convenient/desireable

57

57

Art. I, 8: Power to regulate commerce with foreign nations and among the several states & with the
Indian Tribes.
History

Gibbons v. Ogden Commerce means all phases of business activity, concerning more than one
state. Congress has PLENARY authority over interstate commerce
o This gave Congress authority to regulate commerce at all levels (manufacture, mining,
production, shipping, etc)
1887-1937 Restrictive reading of commerce authority

Commerce authority weakened

Commerce is distinct from mining, manufacturing, and production (E.C. Knight)

States may regulate their own intrastate commerce race to the bottom for regulation

Dagenheart Struck down statute that said father had no right to childs wages
Commerce Power Today

Means of Commerce may be regulated under the commerce clause

Instrumentalities of Commerce may be regulated

Conduct having a substantial effect on commerce may be regulated

(1) Channels highways, railways, airports, canals, etc.


(2) Instrumentalities cars, trucks, planes, trains, etc.
Also includes persons or things cargo or passengers.
(3) Activities, which in the aggregate have a substantial effect on
interstate commerce.
(a) Is it really commerce?

58

58

Commerce: Movement of goods and services


(b) Jurisdictional elements?
(c) Congressional findings?
About the effects on interstate commerce

Means and Instrumentalities

These two prongs are often read together, and they come very close to creating a federal police power

Things that can be regulated


o Use of roads, waterways, airwaves, mail, telephone communication, etc

Example: Congress can make federal crimes out of activities that utilize these means and
instrumentalities.
o Mail and wire fraud statutes make it a federal crime to use the means or instrumentalities of
commerce to transmit fraud.
o Most federal crimes are based on the commerce clause

Note: If the conduct to be regulated utilizes the means and instrumentalities of commerce, it need
not also have a substantial effect on commerce
Conduct having a Substantial Effect on Commerce

Today: Substantial doesnt really mean much

Civil Rights Act of 1964 was passed under this prong (racial discrimination in public accommodations
hurt interstate commerce because it affected travel)

Filburn Growing wheat for personal consumption in violation of a quota can be punished. This was on
the outer-limits of substantial effect prong. If everyone did it, this would be very bad for interstate
commerce.

Gonzalez reaffirmed Filburn. Dealt with growing medical marijuana for personal use.

Lopez Court strikes down a commerce clause regulation. Gun free school zones has nothing to do
with the means and instrumentalities of commerce.

US v. Morrison Court strikes down federal torts provision of Violence Against Women Act. Claim that
gender violence has a substantial effect on interstate commerce was flimsy at-best. Court will strike
down a statute if it is not truly based on commerce

Sebelius Commerce power does not extend to compelling people to enter into commerce (upheld
under taxing power instead)
Lopez Factors:
Commerce Clause Today

Almost everything is covered by the commerce clause

U.S. v. Morrison
Modern courts determine if Congress has exceeded their powers under the Commerce Clause by
utilizing the Lopez Test.
(1) Channels highways, railways, airports, canals, etc.
(2) Instrumentalities cars, trucks, planes, trains, etc.
o Also includes persons or things cargo or passengers.
(3) Activities, which in the aggregate have a substantial effect on interstate commerce.
o (a) Is it really commerce?
Commerce: Movement of goods and services
o (b) Jurisdictional elements?
o (c) Congressional findings?
About the effects on interstate commerce
Here, dealing with the Violence Against Women Act, which gave a victim an action for civil damages.
Not dealing with 1 or 2.
Is it really commerce? No. Gender-based violence.
Was there a jurisdictional element? No. Plaintiff did not have to show that defendant moved
her across state lines, etc.

59

59

Congressional findings? Deterring people from traveling interstate, etc. Too attenuated from
what the Act is meant to bar!
National Federation of Independent Business v. Sebelius (Affordable Care Act)
o The Commerce Power
Differentiates from Wickard: Fundamental difference between telling them not to grow wheat anymore
and telling them to produce more.
Framers gave Congress power to regulate, not to compel commerce.
The power to regulate commerce presupposes the existence of commercial activity to be regulated.
Commerce Clause is about regulating activities, not individuals.
If individuals are to be regulated, theyre going to have to look under something other than the
Commerce Clause.
If young people remain passive and do not buy health insurance, their passivity, in the aggregate, has a
substantial effect on interstate commerce.
Would that means it fits with Wickard and the Lopez test?
No doubt that theres a substantial effect young not in the pool, so middle-aged and elderly
have to pay more.
Bolstered by the Necessary and Proper Clause.
o Necessary and proper means to force you into the market for purposes of keeping
costs low for middle-aged/elderly.
Commerce Clause assumes that there is interstate commerce, and then Congress can regulate that which
exists already.
Government of limited power v. government of plenary powers
Commerce power is to regulate goods and service in interstate commerce, including
individuals incident to that.
Cannot regulate people as people thats police power and reserved to the states.
o Necessary and Proper Clause
Dont actually have to have movement across state lines look at the aggregate.
BUT it is not a power on its own simply a mean to effectuate powers prior to that clause.
The government argued that under the N&P clause, the purpose was a comprehensive health care
regulating scheme, and Congress can use any reasonable means to do so.
Reasonable means to bring about a valid purpose.
Roberts disagrees.
o Has to be a purpose that the Congress has a power to bring about.
o If they dont have the commerce power, then they cant do it.

60

60

Taxing Power

Art. I, 8: Congress shall have the power to lay and collect taxes, duties, imposts to pay the debts
and provide for the common defense and general welfare of the United States.

Sixteenth Amendment: Congress shall have power to law and collect taxes on incomes, from
whatever sources derived, without apportionment among the several states.

Uniform Taxation
o Taxing Power requires congress to tax all states the same way. All ports must be taxed equally,
all states must be taxed equally on applicable taxes
o Sixteenth Amendment: Allows government to tax based on individual income, rather than
taxing the states as a whole or taxing each person equally

Sonzinsky Taxes intended to regulate behavior are constitutionally permissible

61

61

o
o

US v.
o
o
o

Held: So long as a tax raises some form of revenue, it is constitutional, even if it is intended to
regulate behavior
Sebelius Reaffirms holding. Even though failure to have health insurance was penalized, it
was still considered a tax because it raised some form of revenue, and because there was no
fault assigned.
Ptasynski Normally, taxation of states must be uniform (exception)
Oil shortage leads to high prices, and Congress increased taxes
Congress taxed Alaska oil less because of high cost of production in Alaska
Held: Lack of uniformity in taxing was okay because it took into account the increased cost of
production

Spending Power

Art. I, 8: Congress shall have the power to provide for the common defense and general welfare
of the United States.

Simple Explanation Congress may appropriate money to pay for something

Mandate Legislation
o Congress can create grants to give money to the states (block grants)
o Congress can condition receipt of the grants on the state doing something

Often, the state must pass a law to receive the money


o This allows Congress to regulate activities within the province of the state, by forcing the state
to regulate in a certain way in order to obtain funding

South Dakota v. Dole


o Test (is exercise of spending authority proper?)

1. The pursuit (mandate) must be fore the general welfare

The condition must relate to the general welfare

Congress has broad discretion to make such determinations

Drafting requires a recitation of the general welfare in the bill

2. Congress must establish funding conditions unambiguously

The mandate must be drafted clearly

3. The funding condition must be related to the federal interest in particular programs
or projects

Money must be related to the mandate

Ex: Legal drinking age attached to funding for highway repairs

62

62

4. Exercise must not violate another constitutional provision

The Constitution must be read as a whole

Mandate cannot infringe on constitutional liberties

Ex: Child Internet Protection Act

Congress gives money to libraries

Libraries required to put filters on computers to prevent access to


pornography

Struck down as violating 1st Amendment, because adults are permitted


to access pornography (protected speech)

Coercion
o Coercion in the use of the spending power is not unconstitutional
General Welfare US v. Butler: The term is within the discretion of Congress, and it is not tied to
another other power
Other example of spending power: Title IX Schools dont get money if they dont have gender
equality in sports

Property Power

Art. I, 8, Cl. 17: Congress has authority to administer Washington DC

Art. IV, 3, Cl. 2: Congress shall have the power to dispose of and make rules respecting the
territory or other property belonging to the United States
o A lot of western states hate this point because so much land is used for federal parks

Kleppe v. New Mexico


o Federal property power under Article IV is plenary. It operates as if there was no state
authority.
o Federal law prevails on federal property.
Additional Powers

Fiscal Powers
o Art. I, 8, Cls. 2 and 5:

Congress shall have the power to borrow money on the credit of the United States.

Congress shall have the power to coin money, regulate the value thereof, and of
foreign coin, and fix the standard of weights and measures.
o Congress is not constitutionally obligated to balance the budget Has unlimited power to
borrow money.

63

63

Naturalization
o Art. I, 8, Cl. 4: Congress shall have the power to establish a uniform rule of naturalization.
o Immigration is under congressional authority, not state authority
Admiralty
o Art. III, 2, Cl. 1: The judicial power shall extend to all cases of admiralty and maritime
jurisdiction.
o Admiralty is the law of the sae

This creates admiralty jurisdiction

Less important today because waterways are regulated by the commerce clause

Tenth Amendment

The power not delegated to the US by the Constitution, nor prohibited by it to the respective states,
are reserved to the States respectively, or to the people.

Debate Does 10th Amendment carry enforcement powers (brake on the federal government), or is
it merely a reminder to Congress not to overstep its bounds?
o 1792 1880 Mere reminder not to overstep bounds
o 1880 1937 Independent reservation of power to the states; Enforceable restriction on
government power
o 1937 Present Seen simply as a reminder to Congress, not an enforceable restriction

Exception: Printz

Printz v. US Gun Control Act required local sheriffs to perform background checks,
but did not provide funding to do it

Held: Commandeering state officials like this is not permitted. Additionally,


Congress was delegating federal executive authority to state officials

New York v. US (nuclear waste case)


o Facts: Congress forced states to take title to any nuclear waste within their borders after Jan.
1, 1996

Congress said states are liable for any harm caused by the waste within their borders
after that date
o Held

Congress cannot commandeer the states like this. If Congress attached funding
(mandate legislation), then this would have been permissible under the Commerce
Clause

Reno v. Condon
o Congress can regulate public entities (states) using the commerce clause. Additionally, no
violation of 10th Amendment because the statute at issue forbade the states from acting,
rather than forcing it to act.

64

64

Congressional Power to Regulate Private Conduct (Civil War Amendments)

13th Amendment Neither slavery nor involuntary servitude shall exist.


o Congress shall have the power to enforce by appropriate legislation
o 13th Amendment is a unique exception, in that it proscribes private conduct, and Congress
may constitutionally regulate private conduct that violates the 13th Amendment
o Cases Challenging Civil Rights Act of 1883

Held: 13th Amendment allows Congress to legislate against private conduct that
imposes slavery or the badges and incidents of slavery.
o Jones v. Alfred H. Mayer

Held: Congress may rationally determine whether something is, or is not, a badge or
incident of slavery

Rational basis test Congress may only be reversed if it acted irrationally


o Note: 13th Amendment also applies to things like wage slavery

14th Amendment
o Cases Challenging Civil Rights Act of 1883 14th Amendment requires state action
o Congress may only enforce the 14th Amendment against state action

11th Amendment

This amendment does not confer sovereign immunity. It simply means that a state cannot be sued in
federal court without its consent. However, a plaintiff that raises a federal claim may be heard in state
courts.

Chism v. Georgia 11th Amendment ratified to overrule this decision


o Citizen of SC filed suit against Georgia, and Georgia refused to show up. Default judgment.
Georgia claimed it couldnt be sued without its permission. SCOTUS said Article III allowed the
suit. 11th Amendment overruled this decision.
Congressional Power to Interpret Constitution or Overrule SCOTUS

City of Boerne v. Flores


o Held: Congress cannot overrule a SCOTUS constitutional decision with a statute

To what extent may Congress legislate on issues not decided by SCOTUS?


o Is Amendment 14, 5 legislation limited to simply expanding on rights the Court has found?
(No)
o May Congress act beyond those decision of SCOTUS in a way that does not contradict SCOTUS?
(Yes)
o Test Congruence and Proportionality (14 Amend., 5)

Is the Congressional Act congruent and proportional to what the Supreme Court has
done in the past?
Summary of Congressional Power

13th Amendment
o Applies to private and public conduct
o Applies to wage slavery
o Congress has the power to rationally determine what constitutes a badge or incident of
slavery

14th Amendment
o Congress may not use its enforcement power to overrule a SCOTUS decision on the
Constitution
o Congress may pass legislation that enforces a holding of SCOTUS
o Congress may pass legislation on matters SCOTUS has not directly ruled on, so long as the
legislation is congruent and proportional to existing SCOTUS holdings

Often requires drawing analogies analogize the statute to past court decisions

15th Amendment
o Unknown whether the congruence test applies to 15th Amendment

65

65

Last case on this Amendment was City of Rome held that plaintiffs had to show disparate
impact

o
Congress has broad plenary authority under the commerce clause

Dormant Commerce Clause: State and local laws may not unduly burden interstate commerce

Under this situation, the questions are:


o To what extent may states legislate on matters that Congress has not legislated about, and
may never legislate about?
o When can the states legislate, and what restraints are there?
o If Congress has legislated about an issue, and the state tries to legislate on the same matter,
that becomes a question of pre-emption (supremacy clause)

Purpose
o Prevent protectionist behavior where a state favors its own commerce over that of other states

66

66

The Undue Burden Test (a balancing test)

1. Has there been unreasonable discrimination against out-of-staters (in favor of in-staters), or was
there similar treatment?

2. What was the state or local governments legislative purpose in passing the law?
Application of the Test

Do the state laws benefits outweigh the laws burdens on commerce?

If the state law discriminates against out-of-staters, there is a strong presumption the law is
unconstitutional (must achieve important or even compelling government interest)

If the statute is non-discriminatory, courts engage in a balancing process


o Presumption in favor of upholding the law. Law struck down only if the burdens on interstate
commerce outweigh the benefits
State Taxation on Interstate Commerce

To what extent may states tax businesses involved I interstate commerce?

Complete Auto Transit v. Brady


o Test

Significant connection to taxing state

Business must be connected to the state in some way

Fair apportionment

Since every state could tax the business, no state may tax in a way that would
create an undue burden if everyone taxed the business; must tax based on
their apportionment of business

Non-discriminatory

Fair relation to state services

The tax must be related to how the services the business uses in the state
State Quarantine and Inspection

Quarantines of Goods and Products (not people)

State police power allows state to protect health and safety of citizens

67

67

Quarantine cannot be a subterfuge for protecting in-state businesses


Mintz Statute was a good faith quarantine done for public safety; it was not unreasonably
discriminatory; state put forward enough evidence to show they were making a reasonable effort to
stop a real problem
So long as the measure is shown to be a bona fide public health measure, the state can
quarantine
Baldwin State penalized people who bought milk at a lower price from another state
o Held: Struck down as protectionist, and not a bona fide public health measure

Requirements to use local businesses

Almost always struck down because of their protectionist nature

Pike v. Bruce Church, Inc This Court has viewed with particular suspicion state statutes requiring
business operations to be performed in the home state that could more efficiently be performed
elsewhere.
Law Limiting Access to In-State Resources

Pennsylvania v. West Virginia WV passed statute saying in-state gas companies cant export until all
in-state entitled have their needs satisfied
o Held: The state cannot regulate in a way that discriminates between in-state users and out-ofstate users this is unauthorized protectionism
Preserving State-Owned Resources for In-State Use

Market Participation by the State


o Reeves v. Stake

Facts: South Dakota owned a cement company, and charged higher prices to out-ofstate purchasers.

Held: Market Participant Exception Applies

Since the state owns the company, the state can be in the business to serve its
own taxpayers

So long as the state is not acting as a regulator, but acting as a participant, the
state can discriminate
o This is the basic principle of universities charging out-of-state tuition acting as a market
participant
Overruling a SCOTUS decision

If SCOTUS holds that the dormant commerce clause prevents a state from discriminating in its
regulation of commerce, Congress can overrule that decision by passing legislation that permits the
state to discriminate in that fashion.

68

68

o Congress has the authority to allow states to discriminate in interstate commerce


o
Article IV, 2 This Constitution and the laws of the US and treaties shall be the supreme law of the
land, and the judges in every state shall be bound thereby.

Blackletter Law: Federal law prevails over inconsistent/conflicting state laws

Preemption cases generally end with Congress prevailing


When does preemption become an issue?

Common Scenarios
o Congress has not acted, but a state has

The question is whether the state can prevail in the face of congressional silence, when
congress could legislate on this area, or has legislated in an analogous area
o Congress and a state have both acted

The questions that arise is whether the state is simply supporting what congress does,
and if its legislating something different, are the two enactments congruent or in
opposition?
o Voter referenda and initiatives (this is the one that gets all of the attention in the media)

In many states, proposed statutory or constitutional provisions can be put to the voters

Those statutes and provisions can conflict with federal law


Two Types of Preemption

Both types depend on congressional intent (intent to preempt state authority)

Express preemption
o Occurs when Congress, in the statute, expressly says that states may not act in the given
area
o Example: No state shall enact or enforce any law, rule, etc. relating to rates or services of any
airline carrier

Morales v. TWA State passed advertisement regulations. Held: Expressly preempted


as related to rates.
o It is almost impossible to make this litigation-proof. Cases have to interpret the preemption
clause to determine the legislative intent behind the clause.

Implied Preemption
o Occurs when Congress remains silent, but the Court decides that the congressional purpose
cannot be satisfied if the states legislate in the area as well

Congress does not expressly preempt, but the Court believes there was intent to
preempt.

69

69

Types of Implied Preemption

Field Preempion

Conflict Preemption

State law impede full achievement of federal objective


Field Preemption

Question: Whether or not a field of law is preserved to the federal government

Types:
o Congress has legislated so much and so pervasively in a given field that there is no room for
state activity to legislate
o Congress has occupied the field

Example: Foreign policy and foreign relations (immigration and naturalization), bankruptcy, etc

Hines v. Davidowitz By express provision of the Constitution and by Legislation, Congress occupied
the field of immigration and naturalization
Conflict Preemption

Arises when the federal and state law on a particular matter conflict

Heeding one law results in a violation of the other law

Need to determine whether a conflict actually exists

Claim is made that there is implied federal preemption


o Court must decide whether Congress intended to set a minimum standard that would allow the
states to go further
o OR
o If the Congress intended to set the standard, and not allow states to legislate on the issue
Where state law impedes Full Achievement of a Federal Purpose

Court must determine what the Congressional purpose is, and if the state law impedes the
achievement of that purpose

The field is not reserved, and there is no overt conflict but application of the state law would be an
obstacle to full implementation of federal law.
++++
The Treaty Power

Executive negotiates treaties

Treaties must be ratified by the Senate (2/3 vote)

If treaty is ratified, it has the effect of a statute (sub-constitutional)

If a treaty and statute are inconsistent, the latter in time prevails (normal canon of statutory
construction)

Supremacy Clause affects ability to ratify treaties


o Must comport with the Constitution
o Ratification of a treaty may overrule past statutes and may conflict with state laws

Note: Signing and ratifying a treaty are different things; we sign many treaties, but they dont become
law in the US until ratified by statute)
How can Congress overrule SCOTUS on Supremacy Clause Issue?

Normally, when SCOTUS decides a constitutional issue, that decision can only be overruled by a new
SCOTUS decision, or a constitutional amendment

Much like in Dormant Commerce Clause situations, Congress can amend a law or pass legislation
indicating their intent to preempt or not preempt.

70

70

Preemption is based on congressional intent Congress can pass legislation clearly indicating
its intent, overruling a determination by SCOTUS regarding preemption

o
When is precedent overruled?

Is the earlier decision unworkable?

Is there little or no practical reliance on the prior rule?

Is there an evolution of legal principles?

Have the facts changed?


How are constitutional challenges raised?

71

71

Facial Challenge
o Claim that the statute is unconstitutional on its face
o The statute has no possible constitutional applications
o Example: No person may sell a newspaper within 100 yards of a school building
As Applied Challenge
o Statute may be constitutional when applied to other people in other circumstances, but it is not
constitutional as it was applied to this party.
o Example: City ordinance says it is a misdemeanor to behave in the street in a manner
annoying to other persons

A person arrested for screaming obscenities has no constitutional challenge to the


statute

A person arrested for handing out political leaflets can claim violation of 1st
Amendment
Notes
o More difficult to win facial challenges, because they require a showing that the statute is fatally
flawed
o If an as applied challenge is sustained, the statute remains on the books to be used in the
future (except in 1st Amendment cases)

The Main Tests of Substantive Due Process and Equal Protection

Strict Scrutiny
o Test: Government must show that the challenged classification serves a legitimate compelling
state interest and that the law is narrowly tailored to serve that interest.
o Restrictions resulting in Strict Scrutiny Analysis

Anything that restricts a fundamental right

Examples: Denial/Dilution of right to vote; Interstate migration; Access to courts; other


rights recognized as fundamental
o Suspect Classifications resulting in Strict Scrutiny

Race; Religion; National Origin; Alienage


o Notes

Plaintiff almost always wins

Intermediate Scrutiny
o Test: Government must show that the challenged classification serves a legitimate important
state interest and that the law is substantially related to serving that interest
o Quasi-Suspect Classifications result in Intermediate Scrutiny

72

72


Gender; Non-marital children
Notes

This is primarily applicable to equal protection cases


Rational Basis
o Challenger must show that the challenged law is not rationally related to serving a
legitimate government interest
o Notes

Government almost always win

If it doesnt meet strict scrutiny or intermediate scrutiny qualifications, it gets rational


basis scrutiny
o

Inclusiveness

This question goes to the issue of whether a classification is narrowly tailored or substantially related.

Over-Inclusive A law is over-inclusive if it applies to some people who need not be included for the
government to achieve its legitimate purpose

Under-Inclusive A law is under-inclusive if it does not apply to some people who should be
included for the government to achieve its legitimate purpose.

Most statutes are over- or under-inclusive to some degree.


o Strict Scrutiny: Very harsh on over-inclusive statutes that create restrictions
o Rational Basis: Courts are very lax on the relationship between interest and inclusivity
What makes a legitimate state purpose? (Rational Basis)

The traditional police powers of the state make a good example


o States may take action for the purposes of public health, safety, or morals
Almost any goal not prohibited by the Constitution can be a legitimate state interest

Taxing Power
Congress shall have the power to collect taxes, etc. for common defense and general welfare.
Limitation? No.
U.S. v. Butler: Not a limitation on the power to tax or spend. Left up to Congress as to what
that includes.
Congress decided that the purpose of the ACA is the general welfare.
All bills.revenueshall begin in the House.
This began in the Senate!
Sonzinsky v. U.S.
Revenue raising v. regulatory
Used the taxing power to achieve what they couldnt do under the Commerce Clause.
This is like a Sin Tax not interested in raising revenue, but in getting people to stop doing
it. (U.S. v. Sanchez marijuana tax; U.S. v. Kahringer wagering)
The limit on the taxing power is imposing a penalty.
The government CAN tax passivity.
What characteristics make it looks like a tax?
Collected through the IRS and done through the 1040.
If you dont pay federal income tax, you dont pay the fine. Have to have enough income such
that you actually pay federal income tax.
Going to go up or down depending on deductions and federal income tax liability.
If you dont have much income, then the fine is non-existent.
Drexel Furniture: 3 part test to determine if its a tax or penalty.
(1) Heavy burden? (10%+ of net income)
(2) Knowledge requirement
(3) Enforced by the Department of Labor
No knowledge requirement; enforced by IRS, not HHS; minimal fine. Looks like a tax, not a penalty!
Dissent
If this is a tax, its completely novel. (Not the best argument)
Congress called it a penalty 18 times didnt want to call it a tax because taxes are unpopular.

73

73

The court is who imposed the tax, not the Congress.


All 4 justices dissent together; theyre really mad! Mostly at Roberts
Did Roberts greatly enlarge the taxing power?
The taxing power is greater than the Commerce power, but remedially it is far more narrow.
o Commerce Powers array of punishment is huge. Taxing power is just requiring them
to pay the tax
Like the 13th Amendment reaches deep, but remedy is narrow.
Spending Power
U.S. v. Butler
Steward Machine Co. v. Davis
When Congress spending power becomes coercive on the states, then its unconstitutional.
Motive/temptation v. coercive
o Motive of Congress is for the States to yield to the temptation this is no coercion.
Just because the State takes the money doesnt mean its not revocable.
Helvering v. Davis
Social security
When money is spent for the general welfare, the concept of welfare of the opposite is shaped
by Congress, not the States.
Sabri v. U.S.
Distinguished from the ACA case
Federal crime to accept bribery if you get at least $10,000 in federal funds.
Contractor bribed for something that had nothing to do with the federal money. BUT this
statute was so broad that it included it.
o Third party standing
They uphold the use of spending power!
In for a penny, in for a pound. If youre going in for a little, youre in for it all.
o This was basically the holding in Sabri.
o When federal funds go into a municipal budget, it is hard to see where it is actually,
finally spent.
o Federal interest in preventing the wasting away of federal funds by bribery.
South Dakota v. Dole
Federal highway funds would be taken away unless they changed their drinking ages to 21.
4 part test
o (1) Is it in the general welfare? (Defer to Congress)
o (2) Notice of conditions on states must be clear
o (3) Conditions must be related to the federal interest
o (4) Other Constitutional problems?
Rights trump powers.
Unconstitutional conditions
21st Amendment: Transportation into any state for use therein
Infusion of state prerogative in the 21st Am. that the federal government
cant usurp.
For the ACA, the rule of in for a penny, in for a pound is not the law.
Dont lose money by refusing to go with the ACA.
Majority gives the states a choice most states are going to go with the ACA, but some are
holding back, including Missouri.
Is it coercive?
o Will not tolerate Congress that has a policy with lots of detractors.
o Implementation of federal policy in states.
o Pushback against state officials did not make the policy but get all the blame.
o Fundamentally wrong pushback has to be on Congress.
States have to be given adequate notice and germaneness (there must be a nexus between what
the federal government is doing and the money, etc.)
Roberts looks at Dole

74

74

Was 5% of federal highway money, and only .05% of the South Dakota budget.
Compared to the ACA Medicare withholding would lose all funds. Thats 20% of a
states budget on average. Of that 20%, 50-80% of that is federal money a large
amount. Suggestive of coercion
o What is coercion?
What were they asked to give up?
In Dole, had to pass a law that only allows 21+ to drink.
ACA is a huge deal wont be able to spend limited funds on other things if
they dont accept.
If the States buy into this, its no longer Medicare its completely changed.
Was a dog, now its a cow. Transformation of kind.
Ginsburgs Dissent
o Totally novel has never happened before.
o Percentage game, but which percentages matter? How do we draw a line?
o She thinks its a political question and non-justiciable.
Impossible to draw a line.
Preserving the Dole germaneness and notice tests.
Congressional Powers (Often Over Against State Sovereignty)
State Immunity from Federal Regulation
o New York v. U.S.
Low level radioactive waste
Whipsawing states
Forcing them to regulate at the threat of becoming owner of all of this waste huge liability.
Person responsible is Congress you dont get to regulate and then force states to take the blame.
Congress has to take the pushback of its own regulations.
Regulation without responsibility is unconstitutional.
The 10th Amendment is simply a truism the outer edge of Congressional power stops and
states take over.
o Printz v. U.S.
Brady Handgun Violence Prevention Act
Insistence by the Court that the regulator be fully exposed.
Take Care Clause (Art. II, 3)
Congress passes law, executive enforced it, BUT the country sheriffs are enforcing it.
o Lots of pushback and criticism directed at sheriffs.
o Sheriff doesnt answer to the President cannot order him to enforce it; cant delegate
the Take Care Clause.
The Solicitor General was trying to salvage the Act Yes, Congress used commerce power in excess,
but that should be balanced against the good its doing
Court: NO! This is not right. This is the power and structure of the government. This is
STRUCTURAL limit on Congress.
Concurrences by OConnor and Thomas (pg. 266)
o Residual state power NOT states rights.
o Reno v. Condon how do we reconcile this?
Congress tells the states they cannot sell the data collected by the DMV.
Congress can regulate the states, but they cannot tell states to regulate its citizens.
o
o

Enforcement of Federal Rights in Suits Against State Officers: The Eleventh Amendment
State sovereign immunity means that a state cannot be sued in its own name without its consent. Such immunity from suit
pre-existed the formation of the union and continues to inhere in the federalist system in the Constitutions structure.
11th Amendment recognizes that states generally have sovereign immunity when sued in federal court.
o 11th Amendment confirms, rather than creates, sovereign immunity.
o The scope of sovereign immunity is broader than set out in the 11th Amendment.
o The Bankruptcy Clause (Art. I, 8[4]) is said to be an exception whereby sovereign immunity is waived when a
state joins the union. (Central Va. Comm. College v. Katz)

75

75

Does not apply to claims filed in state court. However, states often have sovereign immunity when sued in their own
courts as a matter of state law.
State sovereign immunity and the 11th Amendment do not apply to claims against local governments or other political
subdivisions of the state. Rather, the immunity applies only to claims against the state and its departments and agencies.
State sovereign immunity and the 11th Amendment apply whether plaintiff is a citizen of the defendant-state or of another
state. (Hans v. Louisiana)
Ex Parte Young
o Neither state sovereign immunity nor the 11th Amendment bars federal claims against state officials for
injunctive and/or declaratory relief.
o Although the official is sued in his own name, the claim must be for actions taken in his official capacity.
o The Court reasoned that a state could never authorize one of its officers to violate the Constitution, therefore the
officer must have been acting on his own.
o The Young Fiction: To be able to enjoin violations of federal law, and to allow federal courts to have a
definitive role in interpreting federal law.
o Only applies to prospective relief, not to damages (Edelman). That future compliance with an injunction will
cost the state money, however, make the claim one for damages (Milliken v. Bradley).
Neither state sovereign immunity nor the 11th Amendment bars a federal claim for damages brought against a state
official in his personal capacity. However, any monetary judgment in these personal capacity suits can be collected from
only the individual, not the state.
A state may waive its sovereign immunity and the 11th Amendment.
o Any such waiver must be clear and knowing.
o A state may waive its immunity from suit in its own courts without waiving 11th Amendment immunity when
sued in federal court.
o May refuse to waive its sovereign immunity when sued in state court, even when the claim is on a federal right
that cannot be pursued in federal court because of the 11th Amendment (Alden v. Maine)
o This may leave a plaintiff with a federal right but no judicial venue to enforce it.
Under its 14th Amendment powers, Congress may abrogate a states sovereign immunity and the 11th Amendment.
o Intent to abrogate must be clear in the text of the statute.
o Congress lacks power under Art. I to abrogate a states 11th Amendment immunity when sued in federal court.
Seminole Tribe v. Florida
o Congress also lacks power under Art. I of the Constitution to abrogate a states sovereign immunity when sued in
state court; and this is so even when the claim is on a federal right that cannot be pursued in federal court
because of the 11th Amendment. (Alden)
o 5 of the 14th Amendment: The Congress shall have the power to enforce, by appropriate legislation, the
provisions of this article.
Can use this to abrogate states sovereign immunity to allow them to be sued in federal courts for
damages.
One state can sue another no sovereign immunity issue.
Ex Parte Young
o Federal judiciary was mad because state courts were hearing federal constitutional rights claims.
o The Young Fiction: Allows individual with a constitutional harm to sue in federal court for injunctive relief.
Fear that state judges are hostile to federal constitutional rights.
Fiction: Can sue a state official in federal court for injunctive relief only under the theory that the state
would have never authorized their behavior, so youre suing them in their personal capacity for
injunctive relief and damages.
If suing in official capacity only injunction.
If suing in personal capacity injunctions and damages.
Sue individuals in order to get the benefit of the fiction.
o Only bars retrospective relief does not bar prospective injunctions which will require spending money.

5 of the 14th Amendment


U.S. v. Morrison
o Violence Against Women Act provided a federal civil remedy for victims of gender-motivated violence
o Plaintiff argued that, because of pervasive bias against victims of gender-motivated violence, state justice
systems failed to protect these victims. Used 5 of the 14th Amendment.

76

76

Rejected by the Court.


The Fourteenth Amendment, by its very terms, prohibits only state action
o Football players did not violate the Constitution
o Breyers Dissent
Congress may use the Enforcement Clause to remedy/punish things that arent necessarily constitutional
violations.
It restricts private actors only by imposing liability for private conduct that is, in the main, already
forbidden by state law.
City of Boerne v. Flores
o Church wanted/needed to expand but the city says no. Church brings action under Religious Freedom
Restoration Act, passed by Congress under the authority of 5 of the 14th Amendment.
o Remedial power
Already a right established focusing on the remedy.
Provide remedies for rights that already exist.
Enforce, but not to interpret what is a constitutional violation. Thats the power of the judiciary.
Remedies: Criminalize it, allow a civil action, tax people, etc.
o Congress cannot create a new right and a remedy for that new right!
o Congruence and Proportionality Test
Here, the RFRA fails as it bounds state and local governments (still okay for federal defendants).
Congruence
Means End
Means is the statute targeting certain behavior.
o State and local governments having discriminatory effects on religious groups by
enforcing their laws, etc. Not discriminating on their face, but in their effect.
Need legislative hearings.
For the RFRA, it was inadequate. Too few victims.
Proportionality
Look at the desired end has to be to protect free exercise rights (no more because that would
be new rights).
Lack of proportionality because it protects more.
Disproportionate because it swept much too broadly.
Kimel v. Florida Board of Regents
o ADEA: Allows lawsuits against state and local governments by victims of age discrimination (injunctive and
compensatory relief).
o Using 5 of the 14th Amendment Florida says it goes beyond remedial power.
o Remedy given in the statute was heightened standard of review.
o Age usually only gets rational basis review. UNCONSTITUTIONAL.
o Gave more than what the 14th Amendment provides creating a new right.
Board of Trustees of the University of Alabama v. Garrett
o Another ADA claim
o 2 constitutional violations
o Suits in federal court by state employees to recover money damages by reason of the states failure to comply
with Title I of the ADA are barred by the 11th Amendment.
o In order to authorize private individuals to recover money damages against the States, there must be a pattern of
discrimination by the states which violated the 14th Amendment, and the remedy imposed by Congress must be
congruent and proportional to the targeted violation.
Not met here
To uphold the Acts application would basically allow Congress to rewrite the 14th Am.
Tennessee v. Lane
o ADA accommodations to assist the disabled.
o Opposite result of Garrett.
o Discriminated against individuals need access to the courthouses.
o Stevens (Majority)
As applied, it is constitutional.

77

77

Criminally accused Due Process Clause is implicated because if he doesnt have access to the
courthouse, he cannot take advantage of his rights.
Equal Protection just gets rational basis review, but here we are dealing with the Due Process Clause.
Relates it the other rights of the Constitution!
Title II of the ADA is permitted only when you can link it with some other constitutional right.
o Remedies required from the states were not unreasonable they just had to make reasonable accommodations to
allow disabled persons to exercise their fundamental rights.
Reasonable, prophylactic measure, reasonably targeted to a legitimate end.
Only goes a little beyond; is in aid of a legitimate power.
Congress had the authority under the 14th Am. to regulate the actions of states to accomplish that end.
o CONSTITUTIONAL
o Scalias Dissent
The flabbier the test, the more power for judges.
They can make the outcome whatever they want it to be too malleable.
Anger is mostly direct at Hibbs
Nevada Dept of Human Resources v. Hibbs
o FMLA: In certain instances for mid-sized/large employers, get 12 weeks of paid leave and get to return to job.
o Passed pursuant to 5 of the 14th Amendment.
o Gender discrimination gets intermediate review.
Extends to men how can Congress protect men under the Equal Protection Clause?
If it doesnt apply to men as well, the employers would hire men in order to not deal with the FMLA.
Not only preventing discrimination, but giving 12 weeks leave goes beyond the Enforcement Clause.
o BUT Hibbs upholds the 12 weeks. Why?
Established that certain prophylactic, remedial provisions are okay.

State Regulation of Commerce (Dormant Commerce Clause)


Introduction
Text of the Constitution
o Art. I, 10: Several Limitations on the States
Cant raise own army and navy
Cant print money or coins
Impairment of Contracts Clause: Not really active today
Only applied to existing contracts did not bear on contracts in the future.
Low standard of review: Only have to have rationale/reasonable purpose for doing so.
o Art. I, 9 & 10: Not imposing duties on interstate goods.
Certain powers were delegated to Congress that the Court has deemed exclusive.
o Examples: Operation of the Postal Service; patent registration
o But the text itself did not say exclusive!
o So you need to research to find out if the Court has said its exclusive or no.
o Arizona v. U.S.
Supreme Court did not say that immigration is an exclusive federal power.
Federal government has occupied most of it, but there is some room for state immigration laws.
Dormant Commerce Clause
Discrimination against interstate commerce?
o (1) Per Se Test: Is it intentionally anti-competitive?

78

78

Yes: Unconstitutional
No: Go to Step 2
o (2) Discriminatory Impact Only?
Balancing Test
Is commerce burdened?
Almost always yes, because otherwise there wouldn't be a suit.
How bad is the effect?
Really bad: Struck down
Not bad: Upheld
If states reason is health or safety, it will be upheld.
Most challenges are going to try to win in Step 1. The balancing test is more rigorous than rational basis review.
Conceptual Problem
o Art. I, 8, 3 Negative Commerce Clause The Congress shall have the power to regulate commerce with
foreign nations, and among the several States
o Commerce power for Congress. Congress could have regulated but has remained silent.
o While this clause is about delegating power, but read into the clause is a right.
Right in industry to sue a State that is regulating interstate commerce detrimentally.
Right to industry not to be harmed by States in interstate commerce.
o Vested a right in industry so industry has standing to sue.
The cases where the state has intentionally or inadvertently created substantial burdens on interstate commerce fall into
types.
o Must first identify very carefully what the states restriction is on commerce.
o Intentional discrimination against interstate commerce
o Implied Restrictions Transportation
Instrumentalities of commerce
Navajo Freight Lines case: Iowa law was struck down because they were the only ones who had the
particular law, so it was a burden on interstate commerce.
o Implied Restrictions of the Commerce Clause Production and Trade
Restricting Importation and Insulating In-State Business From Out-of-State Competition
Mintz v. Baldwin
o Protectionist for cattle producers inside the state of New York
o Purpose: Prevent disease from being brought into the state
o Court: Good enough reason notwithstanding the increase in price for cattle
producers outside of New York, they sustain the law.

Requiring Business Operations to Be Performed in the Home State


Minnesota v. Barber
o MN statute prohibited the sale for human food of fresh meat not taken from animal
inspected in MN within 24 hours before slaughter.
o Defended on the basis of consumer health
o Why is it not a good enough reason here?
Rather than relying on their own inspectors, MN should rely upon the
inspectors of other states as well.
Its okay for MN to require inspection, but they cant require inspection in
MN only.
Foster-Fountain Packing Co. v. Haydel
o LA banned the sale of shrimp that had been hulled in outside states; said that shrimp
had to be hulled in LA.
o The throwaway part of the shrimp was used as feed for chickens, so the pricing of
producing chickens went up as well.
o LA hoarding all of this; getting all of the benefit! No defense for LA that this should
be done.

79

79

Pike v. Bruce Church, Inc.


o Doesnt actually have to accuse of being protectionist because it will clearly fail the
balancing test.
o Cantaloupes grown in AZ with CO river water. AZ branding cantaloupes because they
wanted to make AZ cantaloupes sell as a superior price because consumers would
believe they were of superior quality.
o Unconstitutional.
Requires everyone growing cantaloupes in AZ to open an AZ packaging
plant.
If its an AZ-origin melon being processed in CA, CA workers can put a little
label on the melon that says its from AZ, etc.
Preserving Resources for In-State Consumption
Pennsylvania v. West Virginia
o Natural gas hoarding WV negotiated a deal with a pipeline company; gas will go to
PA if the PA residents will pay more.
o Doesnt fail the per se test, but does fail the balancing test.
In a moment of scarcity, WV is going to get taken care of first at whatever
the going price is.
BUT if theyre willing to pay more in PA, then it will go there first instead.
Contracts to deliver natural gas are long-term, so if theyre going to sign a
contract for delivery, theyre going to have to lock in longer before they
know its going to be scarce, etc.
Preserving State-Owned Resources for In-State Use
EXCEPTION: When a state actually owns the means of production either raw material or
state-owned factory, etc.
o Reeves, Inc. v. Stake
South Dakota owns the factory which produces cement.
Dormant Commerce Clause does not apply because the state is then just
treated as one more character of commerce.
The fact that the state produces it and is selling it at a certain price and only
to SD residents is okay. Doesnt matter because they are the producers so
they can choose to do whatever they want.
Limits on Business Entry
Interstate Mobility of Persons

80

80

Anda mungkin juga menyukai